Site Loader

Содержание

5.4. Момент инерции твердого тела

Чтобы найти момент инерции тела, надо просуммировать момент инерции всех материальных точек, составляющих данное тело

(5.4)

В общем случае, если тело сплошное, оно представляет собой совокупность множества точек с бесконечно малыми массами , и моменты инерции тела определяется интегралом

(5.5)

о где — расстояние от элемента до оси вращения.

Распределение массы в пределах тела можно охарактеризовать с помощью
плотности

(5.5)

где m — масса однородного тела, V — его объем. Для тела с неравномерно распределенной массой это выражение даетсреднюю плотность.

Плотность в данной точке в этом случае определяется следующим образом

и тогда

(5. 6)

Пределы интегрирования зависят от формы и размеров тела Интегрирование уравнения (5.5) наиболее просто осуществить для тех случаев, когда ось вращения проходит через центр тяжести тела. Рассмотрим результаты интегрирования для простейших (геометрически правильных) форм твердого тела, масса которого равномерно распределена по объему.

Момент инерции полого цилиндра с тонкими стенками, радиуса R.

Для полого цилиндра с тонкими стенками

Сплошной однородный диск. Ось вращения является осью диска радиуса . и массы m с плотностью

Высота диска h. Внутри диска на расстоянии вырежем пустотелый цилиндр с толщиной стенки и массой . Для него

Весь диск можно разбить на бесконечное множество цилиндров, а затем просуммировать:

Момент инерции шара относительно оси, проходящей через центр тяжести.

Момент инерции стержня длиной L и массой m относительно оси, проходящей:

а) через центр стержня —

б) через начало стержня —

Теорема Штейнера. Имеем тело, момент инерции которого относительно оси, проходящей через его центр масс известен. Необходимо определить момент инерции относительно произвольно оси параллельной оси . Согласно теореме Штейнера, момент инерции тела относительно произвольной оси равен сумме момента инерции тела относительно оси, проходящей через центр масс и параллельной данной оси, плюс произведение массы тела на квадрат расстояния между осями:

(5.7)

Момент инерции твердого тела

Определение и общие сведения о моменте инерции твердого тела

Это скалярная (в общем случае тензорная) величина.

   

где – массы материальных точек, на которые разбивают тело; на квадраты расстояний от материальной точки до оси вращения.

Для непрерывного однородного тела, вращающегося около оси, момент инерции чаще определяют как:

   

где r – функция положения материальной точки в пространстве; – плотность тела; –объем элемента тела.

Тензор инерции

Совокупность величин:

   

называют тензором инерции. Диагональные элементы тензора: . Тензор инерции является симметричным.

Пусть все недиагональные элементы тензора равны нулю, не равны нулю только диагональные составляющие. Тогда тензор запишем как:

   

В таком случае оси тела совпадают с осями координат и являются главными осями инерции. Величины:

   

называют главными моментами инерции. Тензор в виде (4) приведен у диагональному виду. Моменты инерции, находящиеся вне главной диагонали матрицы (3) называются центробежными. Если оси системы координат направлены вдоль главных осей инерции тела, то центробежные моменты инерции равны нулю.

Если главные оси проведены через центр масс тела, то они называются центральными главными осями, а тензор центральным тензором.

Главные оси не всегда для тела не всегда легко отыскать. Но иногда достаточно использовать соображения симметрии. Так, в шаре относительно любой точки главные оси можно найти так. Одна из главных осей проходит через центр шара, две другие ориентированы произвольно в плоскости, которая перпендикулярна первой оси.

Составляющие момента инерции сплошного тела относительно осей декартовой системы координат определены как:

   

   

   

где – координаты элемента массы тела (), которая обладает объемом .

Момент инерции твердого тела зависит от формы тела и распределения ассы в теле относительно оси вращения.

Величины, равные:

   

называют радиусами инерции тела по отношению к соответствующим осям системы координат.

Теорема Штейнера

В некоторых случаях вычисление момента инерции существенно облегчает знание теоремы Штейнера (иногда ее называют теоремой Гюйгенса): Момент инерции тела (J) относительно произвольной оси равен моменту инерции относительно оси, которая проведена через центр масс рассматриваемого тела (), плюс произведение массы тела (m) на расстояние между осями в квадрате, при условии, если оси параллельны:

   

Примеры решения задач

«Момент инерции абсолютно твердого тела 10 класс»

План урока

Раздел : Динамика

Школа:

Дата:

ФИО учителя:

Класс: 10

Количество присутствующих:

Отсутствующих:

Тема урока

Момент инерции абсолютно твердого тела

Цели обучения, которые достигаются на данном уроке (ссылка на учебную программу)

использовать теорему Штейнера для расчета момента инерции материальных точек

Цели урока

Научится рассчитывать энергию вращательного движения и применять теорему Штейнера для расчета момента инерции абсолютно твердых тел

Критерии оценивания

Учащийся достигли цели обучения если могут рассчитывать энергию вращательного движения и применять теорему Штейнера для расчета момента инерции абсолютно твердых тел

Языковые цели

Развитие навыков говорения и слушания при обсуждении материала.

Лексика и терминология, специфичная для предмета:

энергия вращательного движения, момент инерции тела, теорема Штейнера, второй закон Ньютона для вращательного движения

Привитие ценностей

Коммуникативные навыки, ответственность, аккуратность, расширение кругозора, множественный интеллект, функциональная грамотность

Межпредметные связи

Математика, геометрия

Навыки использования ИКТ

Интерактивная доска, ноутбуки, PowerPoint

Предварительные знания

Кинетическая энергия поступательного движения, основные характеристики вращательного движения, второй закон Ньютона

Ход урока

Запланированные этапы урока

Запланированная деятельность на уроке

Ресурсы

Начало урока 2 мин

5 мин

Вступительное слово учителя (Мобилизующее начало урока. Психологический настрой на урок.)

Проверка выполнения домашнего задания (сочетания контроля, самоконтроля и взаимоконтроля)

Цель: Проверить уровень усвоения учебного материала, выявить пробелы и скорректировать их.

А)Актуализация опорных знаний

Форма: коллективная

Метод: индивидуальный

Прием: вопрос-ответ

Взаимопроверка

Вопросы:

сформулируйте три знаменитые законы движения.

I закон (закон инерции):

Материальная точка (тело) сохраняет состояние покоя или равномерного прямолинейного движения до тех пор, пока воздействие со стороны других тел не заставит её (его) изменить это состояние.

II закон В ИСО ускорение тела пропорционально векторной сумме всех действующих на тело сил и обратно пропорционально массе тела:

III закон Тела действуют друг на друга с силами, равными по модулю и противоположными по направлению:

В) Решения задач (Индивидуальная работа)

1.Человек везет двое связанных саней, прикладывая силу под углом 300 к горизонту. Найдите эту силу, если известно, что сани движутся равномерно. Массы саней по 40кг. Коэффициент трения 0,3.

2.Шайба остановилась через 5с после удара клюшкой на расстоянии 20м от места удара. Масса шайбы 100г. Определить силу трения между шайбой и льдом.

3.Чему должен быть равен минимальный коэффициент трения и между шинами и поверхностью дороги с уклоном 300, чтобы автомобиль мог двигаться по ней вверх с ускорением 0,5м/с2?

Интерактивная доска

Карточки вопросники

Середина урока

15 мин

17 мин

18 мин

  1. Подготовка к активному усвоению и осмысления учебного материала

Цель: Обеспечить мотивацию и принятия учащимися цели учебно-познавательной деятельности.

Форма: ППС и индивидуальная

Метод: поисково-эвристический


А) Рассмотрим вращающееся вокруг закрепленной оси OO′ твердое тело

Скажите пожалуйста обладает ли тело кинетической энергии?

Знакомое нам выражение для кинетической энергии поступательного движения Wк = mv2/2 неудобно для вычисления кинетической энергии вращательного движения.

Чтобы вычислить полную кинетическую энергию вращательного движения, необходимо просуммировать кинетические энергии поступательного движения по всем точкам, т.е.

(1)

Задача упрощается в связи с тем, что угловая скорость вращения ω = ∆ ф/∆t у всех точек одинакова, поэтому линейную скорость υ в выражении (1) заменим на угловую, так как υ = ωR.

Тогда выражения (1) запишем следующим образом


(2)

Сумма, находящаяся в скобках:

1. Эта сумма — скалярная величина.

2. Она зависит от распределения массы тела относительно оси вращения: чем дальше находится масса от оси вращения, тем больше эта сумма.

Вопрос ученикам : У какого тела — тонкого обруча или сплошного диска, одинаковых по радиусу и массе, эта сумма больше.

У обруча, так как все mi-е точки обруча находятся на самом максимальном расстоянии ri = R. Поэтому эта сумма — конкретная величина для данного тела. Она называется

момент инерции тела, обозначается символом J. Следовательно:

(3)

Выражение (3) похоже на уравнение для кинетической энергии поступательного движения, т. е. формула (3) позволяет рассчитать кинетическую энергию вращательного движения. Из нее следует, что обруч и диск, раскрученные до одинаковой угловой скорости, обладают разными кинетическими энергиями.

Моменты инерции некоторых тел, вычисленные относительно оси вращения, проходящей через центр масс

Моменты инерции различных тел правильной геометрической формы.

Момент инерции тонкого стержня относительно оси, проходящей

перпендикулярно к его концу

Обратите внимание, что в примере 6 по таблице дан случай, когда ось вращения не проходит через центр масс. В таких случаях для нахождения момента инерции можно воспользоваться теоремой Гюйгенса — Штейнера:

момент инерции тела относительно любой оси, не проходящей через центр тяжести, равен сумме момента инерции данного тела относительно оси, проходящей через центр тяжести, и произведению массы тела на расстояние между этими осями.

  1. Практическая часть урока (применения теоретические данные в решении задач)

1.Определите момент инерции материальной точки массой 300 г относительно оси, отстоящей от нее на расстоянии 20 см.

(Ответ: 12 г · м2)

2. На концах тонкого однородного стержня длиной 90 см и массой 300 г прикреплены шарики массами 100 г и 200 г. Определите момент инерции этой системы относительно оси, перпендикулярной стержню и проходящей через: а) первый шарик; б) точку, отстоящую от первого шарика на 30 см в) середину стержня.

(Ответ: а) 0,243 кг · м2; б) 108 г · м2; в) 81 г · м2)

3. Определите момент инерции плоской однородной прямоугольной пластинки массой 900 г относительно оси, совпадающей с одной из сторон, если длина другой стороны 20 см.

Конец урока

5 мин

1.Рефлексия

Цель: Мобилизация учащихся на рефлексию своего поведения (мотивации, способов деятельности, общения).

Рефлексия:

— что узнал, чему научился

— что осталось непонятным

— над чем необходимо работать

2. Домашнее задание.

Учебник Б.А.Кронгарт, стр. 73,

Упр.5

  1. Итог урока

Цель: Дать анализ и оценку успешности достижения цели и наметить перспективу последующей работы.

Стикеры

Дифференциация – каким образом Вы планируете оказать больше поддержки? Какие задачи Вы планируете поставить перед более способными учащимися?

Оценивание – как Вы планируете проверить уровень усвоения материала учащимися?

Здоровье и соблюдение техники безопасности

Все учащиеся:

знают момент инерции тела

Большинство учащихся будут:

Применять теоретические навыки полученные на уроке при решении задач на доске

Некоторые учащиеся смогут:

Применять самостоятельно теоретические навыки полученные на уроке при решении задач

Используйте данный раздел для записи методов, которые Вы будете использовать для оценивания того, чему учащиеся научились во время урока.

Здоровье сберегающие технологии.

Используемые физминутки и активные виды деятельности.

Пункты, применяемые из Правил техники безопасности на данном уроке.

Рефлексия по уроку

Были ли цели урока/цели обучения реалистичными?

Все ли учащиеся достигли ЦО?

Если нет, то почему?

Правильно ли проведена дифференциация на уроке?

Выдержаны ли были временные этапы урока?

Какие отступления были от плана урока и почему?

Используйте данный раздел для размышлений об уроке. Ответьте на самые важные вопросы о Вашем уроке из левой колонки.

Общая оценка

Какие два аспекта урока прошли хорошо (подумайте как о преподавании, так и об обучении)?

1:

2:

Что могло бы способствовать улучшению урока (подумайте как о преподавании, так и об обучении)?

1:

2:

Что я выявил(а) за время урока о классе или достижениях/трудностях отдельных учеников, на что необходимо обратить внимание на последующих уроках?

Новосибирский государственный архитектурно-строительный университет — Сибстрин

«Городское строительство» – профиль подготовки, востребованный в любом мегаполисе

Новосибирский государственный архитектурно-строительные университет (Сибстрин) приглашает абитуриентов на современную и востребованную образовательную программу Городское строительство направление 08.03.01 Строительство. Специалисты данного направления имеют обширные знания в вопросах планирования и проектирования городских пространств, зданий, сооружений и их комплексов, требований современного градостроительного законодательства, экологических особенностей развития застроенных территорий. Они умеют осуществлять весь цикл строительства, начиная с этапа идей до технической эксплуатации возведенного объекта. Подготовка бакалавров предполагает изучение как точных инженерных, так и творческих дисциплин.

Приглашаем абитуриентов на направление «Теплогазоснабжение и вентиляция», выпускников которого ждут крупнейшие предприятия отрасли: СГК и Неватом

Подготовка в НГАСУ (Сибстрин) будущих специалистов по профилю «Теплогазоснабжение и вентиляция» дает им возможность трудиться на стыке трех мощнейших комплексов народного хозяйства России – строительного, топливно-энергетического и жилищно-коммунального, что делает эту специальность уникальной. Компетенции специалистов, обучающихся по данному профилю, позволяют им проектировать, производить монтаж и наладку, эксплуатировать системы жизнеобеспечения современных зданий и сооружений: теплоснабжения и горячего водоснабжения, газоснабжения, отопления и вентиляции, кондиционирования воздуха, теплогенерирующие установки, системы очистки вентиляционного воздуха, системы учета и рационального потребления энергоресурсов. Объектами профессиональной деятельности специалистов по профилю «Теплогазоснабжение и вентиляция» являются: тепловые и газовые сети, системы вентиляции…

Ждем абитуриентов на IT-направление!

НГАСУ (Сибстрин) на протяжении последних 20 лет занимает лидирующие позиции в подготовке конкурентоспособных IT-специалистов, отвечающих требованиям рынка труда и обладающих востребованными компетенциями в сфере информационных технологий. Направление «Информационные системы и технологии» входит в перечень специальностей и направлений подготовки высшего образования, соответствующих приоритетным направлениям модернизации и технического развития российской экономики. Качество подготовки наших специалистов подтверждено независимой оценкой качества высшего образования (НОКВО).

§16. Момент инерции твердого тела. Абсолютно твердое тело.

Кроме материальной точки и системы материальных точек в классической механике рассматривают твердые тела. Как правили, в этом случае выделение отдельных материальных точек не производят. Твердое тело рассматривают как единое целое. Естественно, что для его описания вводят новые понятия. Например, вместо массы удобнее рассматривать момент инерции. Как и масса, момент инерции это скалярная и аддитивная величина. Но она не имеет смысла, если не указана ось, относительно которой определяется этот момент.

Проще всего момент инерции ввести для одной материальной точки m(см. рис. 3).

Рисунок 3 показывает, что момент инерции определяется (в данном случае) относительно оси ОО. Если материальная точка находится на расстоянии Х от оси, то момент инерции определяется формулой

.

Если же взять другую ось, то, естественно, расстояние материальной точки Х1до новой оси буден иным, а значит, изменится и момент инерции.

Если имеется несколько материальных точек, то рисунок 3 обобщается, давая для полного момента инерции выражение

.

Это равенство записано при определенном выборе оси и выражает свойство аддитивности моментов инерции (т.е. возможность сложения моментов инерции отдельных элементов).

Два этих определения позволяют найти моменты инерции простых твердых тел. Например, момент инерции обруча, радиус которого равен , а масса равна,относительно оси, перпендикулярной плоскости обруча и проходящей через его центр, равен (в силу того, что все элементы обруча находятся на одинаковых расстояниях от оси) величине

.

Эта же формула определяет момент инерции пустотелого цилиндра относительно оси его симметрии.

Приведет два примера вычисления моментов инерции более сложных твердых тел.

1. Сплошной диск или сплошной цилиндр. Ось по-прежнему перпендикулярна плоскости диска и проходит через его центр. Выделим коаксиальное кольцо радиусом и толщиной. Площадь кольца равнаи на него приходится масса

.

Соответственно этому момент инерции выделенного кольца равен

.

Полный момент инерции диска получим, проведя интегрирование поот нуля до. Легко видеть, что после интегрирования и сокращения на множитель, результатом является

.

В силу симметрии этот же результат пригоден для сплошного цилиндра любой длины, относительно оси, совпадающей с осью цилиндра.

2. Пусть имеется однородный стержень массой и длиной(см. Рис. 4)

Выберем на оси интервал значений и определим момент инерции массы, соответствующей этому интервалу. Эта масса равна

,

а соответствующий элемент момента инерции

.

Полный момент инерции равен интегралу от в пределах отдо(т.е. по области, где масса отличается от нуля). Вычисление интеграла дает

.

Нам важны два частных случая:

А). a= 0; тогда.

Б). ; тогда.

Заметим, что случай А) соответствует оси, перпендикулярной стержню и проходящей через его конец, тогда как случай Б) соответствует оси, проходящей через центр масс стержня. Сопоставление двух этих случаев дает

.

Последнее соотношение – частный случай теоремы Штейнера:момент инерции твердого тела относительно произвольной оси равен моменту инерции относительно оси, проходящей через центр масс тела параллельно заданной оси, плюс произведение массы тела на квадрат расстояния между осями.Эта теорема позволяет переходить от моментов инерции простых систем к более сложным. Например, момент инерции сплошного диска относительно оси, походящей через его край перпендикулярно плоскости диска равен.

Заметим еще, что абсолютно твердым телом называют тело, момент инерции которого постоянен (не зависит от времени). Ведро воды, например, нельзя считать абсолютно твердым телом.

определение, законы, формулы для чайников

Часто мы слышим выражения: «он инертный», «двигаться по инерции», «момент инерции». В переносном значении слово «инерция» может трактоваться как отсутствие инициативы и действий. Нас же интересует прямое значение.

Ежедневная рассылка с полезной информацией для студентов всех направлений – на нашем телеграм-канале.

Что такое инерция

Согласно определению инерция в физике – это способность тел сохранять состояние покоя или движения в отсутствие действия внешних сил.

Если с самим понятием инерции все понятно на интуитивном уровне, то момент инерции – отдельный вопрос. Согласитесь, сложно представить в уме, что это такое. В этой статье Вы научитесь решать базовые задачи на тему «Момент инерции».

Определение момента инерции

Из школьного курса известно, что масса – мера инертности тела. Если мы толкнем две тележки разной массы, то остановить сложнее будет ту, которая тяжелее. То есть чем больше масса, тем большее внешнее воздействие необходимо, чтобы изменить движение тела. Рассмотренное относится к поступательному движению, когда тележка из примера движется по прямой.

 

По аналогии с массой и поступательным движением момент инерции – это мера инертности тела при вращательном движении вокруг оси.

Момент инерции – скалярная физическая величина, мера инертности тела при вращении вокруг оси. Обозначается буквой J и в системе СИ измеряется в килограммах, умноженных на квадратный метр.

Как посчитать момент инерции? Есть общая формула, по которой в физике вычисляется момент инерции любого тела. Если тело разбить на бесконечно малые кусочки массой dm, то момент инерции будет равен сумме произведений этих элементарных масс на квадрат расстояния до оси вращения.

Это общая формула для момента инерции в физике. Для материальной точки массы m, вращающейся вокруг оси на расстоянии r от нее, данная формула принимает вид:

Теорема Штейнера

От чего зависит момент инерции? От массы, положения оси вращения, формы и размеров тела.

Теорема Гюйгенса-Штейнера – очень важная теорема, которую часто используют при решении задач.

Кстати! Для наших читателей сейчас действует скидка 10% на любой вид работы

Теорема Гюйгенса-Штейнера гласит:

Момент инерции тела относительно произвольной оси равняется сумме момента инерции тела относительно оси, проходящей через центр масс параллельно произвольной оси и произведения массы тела на квадрат расстояния между осями.

Для тех, кто не хочет постоянно интегрировать при решении задач на нахождение момента инерции, приведем рисунок с указанием моментов инерции некоторых однородных тел, которые часто встречаются в задачах:

 

Пример решения задачи на нахождение момента инерции

Рассмотрим два примера. Первая задача – на нахождение момента инерции. Вторая задача – на использование теоремы Гюйгенса-Штейнера.

Задача 1. Найти момент инерции однородного диска массы m и радиуса R. Ось вращения проходит через центр диска.

Решение:

Разобьем диск на бесконечно тонкие кольца, радиус которых меняется от 0 до R и рассмотрим одно такое кольцо. Пусть его радиус – r, а масса – dm. Тогда момент инерции кольца:

Массу кольца можно представить в виде:

Здесь dz – высота кольца. Подставим массу в формулу для момента инерции и проинтегрируем:

В итоге получилась формула для момента инерции абсолютного тонкого диска или цилиндра.

Задача 2. Пусть опять есть диск массы m и радиуса R. Теперь нужно найти момент инерции диска относительно оси, проходящей через середину одного из его радиусов.

Решение:

Момент инерции диска относительно оси, проходящей через центр масс, известен из предыдущей задачи. Применим теорему Штейнера и найдем:

Кстати, в нашем блоге Вы можете найти и другие полезные материалы по физике и решению задач.

Надеемся, что Вы найдете в статье что-то полезное для себя. Если в процессе расчета тензора инерции возникают трудности, не забывайте о студенческом сервисе. Наши специалисты проконсультируют по любому вопросу и помогут решить задачу в считанные минуты.

Момент инерции | Физика для студентов | Студенту | Статьи и обсуждение вопросов образования в Казахстане | Образовательный сайт Казахстана

При вращении твердого тела вокруг неподвижной оси отдельные точки тела описывают окружности, центры которых лежат на оси вращения. Основы кинематики вращательного движения были изложены в разделе 1.5.

Для описания вращательного движения твердого тела вводят понятие момента инерции.

Моментом инерции I материальной точки называется скалярная физическая величина, определяемая произведением ее массы mна квадрат радиуса окружности R, по которой она может двигаться относительно некоторой произвольно выбранной оси ОО‛ (рис.4.1,а):

I=mR2


Рис.4.1. К определению понятия момента инерции

Если твердое тело, вращающееся относительно некоторой произвольно выбранной оси ОО’, представить в виде системы материальных точек массой dm и просуммировать моменты инерции этих так называемых элементарных масс, то получим момент инерции всего тела:

где ri – радиус вращения i–й элементарной массы, а интеграл берется по всему объему тела (рис. 4.1,б).

Для однородных тел, для которых плотность ρ=m/V (где m – масса тела, а V – его объем, т.е. плотность определяется массой, заключенной в единице объема), момент инерции будет вычисляться по формул:

I=∫r2dm=∫r2pdV=p∫r2dv, т.е. I=ρr2dV.

Ниже приведены значения моментов инерции для некоторых однородных тел правильной формы с массой m относительно оси, проходящей через центр масс тела.

Таблица 2

Моменты инерции тел правильной формы

Тело
Положение оси вращения
Момент инерции
Полый тонкостенный цилиндр радиусом R
Ось симметрии
mR2
Сплошной цилиндр или диск радиусом R
Ось симметрии
0.5*mR2
Прямой тонкий стержень длиной l
Ось перпендикулярна стержню и проходит через его середину
1/12*ml2
Шар радиусом R
Ось проходит через центр шара
2/5*mR2

Для определения момента инерции тела относительно произвольной оси используется теорема Штейнера.

Теорема Штейнера: если известен момент инерции тела относительно оси ОО’, проходящей через центр масс тела (обозначим его Io), то момент инерции тела относительно любой параллельной ей оси ZZ’ (обозначим его I) равен:

I=Io+md2,

где m – масса тела; d – расстояние между осями (рис.4.2).


Рис.4.2. К теореме Штейнера

механиков | Определение, примеры, законы и факты

механика , наука, изучающая движение тел под действием сил, включая особый случай, когда тело остается в покое. В первую очередь проблема движения — это силы, которые тела действуют друг на друга. Это приводит к изучению таких тем, как гравитация, электричество и магнетизм, в зависимости от природы задействованных сил. Учитывая силы, можно искать способ, которым тела движутся под действием сил; это предмет собственно механики.

Британская викторина

Викторина «Все о физике»

Кто был первым ученым, проведшим эксперимент по управляемой цепной ядерной реакции? Какая единица измерения для циклов в секунду? Проверьте свою физическую хватку с помощью этой викторины.

Исторически механика была одной из первых возникших точных наук.Его внутренняя красота как математической дисциплины и ранний замечательный успех в количественном учете движений Луны, Земли и других планетных тел оказали огромное влияние на философскую мысль и послужили толчком для систематического развития науки.

Механику можно разделить на три части: статика, которая имеет дело с силами, действующими на покоящееся тело и в нем; кинематика, описывающая возможные движения тела или системы тел; и кинетика, которая пытается объяснить или предсказать движение, которое произойдет в данной ситуации.В качестве альтернативы механику можно разделить по типу изучаемой системы. Простейшей механической системой является частица, определяемая как настолько маленькое тело, что его форма и внутренняя структура не имеют значения в данной задаче. Более сложным является движение системы из двух или более частиц, которые действуют друг на друга и, возможно, испытывают силы, действующие со стороны тел вне системы.

Принципы механики были применены к трем общим областям явлений.Движение таких небесных тел, как звезды, планеты и спутники, можно предсказать с большой точностью за тысячи лет до того, как они произойдут. (Теория относительности предсказывает некоторые отклонения от движения в соответствии с классической или ньютоновской механикой; однако они настолько малы, что их можно наблюдать только с помощью очень точных методов, за исключением задач, затрагивающих всю или большую часть обнаруживаемой Вселенной. Как вторая область, обычные объекты на Земле вплоть до микроскопических размеров (движущиеся со скоростью намного ниже скорости света) правильно описываются классической механикой без значительных исправлений.Инженер, проектирующий мосты или самолеты, может с уверенностью использовать ньютоновские законы классической механики, даже если силы могут быть очень сложными, а вычислениям не хватает прекрасной простоты небесной механики. Третья область явлений включает поведение материи и электромагнитного излучения в атомном и субатомном масштабах. Хотя вначале были достигнуты ограниченные успехи в описании поведения атомов в терминах классической механики, эти явления должным образом рассматриваются в квантовой механике.

Получите подписку Britannica Premium и получите доступ к эксклюзивному контенту. Подпишитесь сейчас

Классическая механика занимается движением тел под действием сил или равновесием тел, когда все силы уравновешены. Этот предмет можно рассматривать как разработку и применение основных постулатов, впервые сформулированных Исааком Ньютоном в его книге Philosophiae Naturalis Principia Mathematica (1687), широко известной как Principia . Эти постулаты, называемые законами движения Ньютона, изложены ниже.Их можно использовать для предсказания с большой точностью самых разных явлений, от движения отдельных частиц до взаимодействий очень сложных систем. В этой статье обсуждается множество этих приложений.

В рамках современной физики классическую механику можно понять как приближение, вытекающее из более глубоких законов квантовой механики и теории относительности. Однако такой взгляд на место объекта сильно недооценивает его важность в формировании контекста, языка и интуиции современной науки и ученых.Наш современный взгляд на мир и место человека в нем прочно укоренен в классической механике. Более того, многие идеи и результаты классической механики выживают и играют важную роль в новой физике.

Центральными понятиями классической механики являются сила, масса и движение. Ни сила, ни масса не были четко определены Ньютоном, и оба они были предметом многих философских спекуляций со времен Ньютона. Оба они наиболее известны своими эффектами. Масса — это мера склонности тела сопротивляться изменениям в состоянии движения.С другой стороны, силы ускоряют тела, то есть они изменяют состояние движения тел, к которым они приложены. Взаимодействие этих эффектов — основная тема классической механики.

Хотя законы Ньютона фокусируют внимание на силе и массе, три другие величины приобретают особое значение, потому что их общее количество никогда не меняется. Эти три величины — энергия, (линейный) импульс и угловой момент. Любой из них может быть перемещен из одного тела или системы тел в другое.Кроме того, энергия может менять форму, будучи связанной с единственной системой, проявляясь как кинетическая энергия, энергия движения; потенциальная энергия, энергия позиции; тепло или внутренняя энергия, связанная со случайными движениями атомов или молекул, составляющих любое реальное тело; или любая комбинация из трех. Тем не менее полная энергия, импульс и угловой момент во Вселенной никогда не меняются. Этот факт выражается в физике, говоря, что энергия, импульс и угловой момент сохраняются.Эти три закона сохранения вытекают из законов Ньютона, но сам Ньютон их не выражал. Их нужно было обнаружить позже.

Примечательно, что, хотя законы Ньютона больше не считаются фундаментальными и даже не совсем правильными, три закона сохранения, вытекающие из законов Ньютона — сохранение энергии, импульса и момента количества движения — остаются в точности верными даже в квантовая механика и теория относительности. Фактически, в современной физике сила больше не является центральным понятием, а масса — лишь одним из множества атрибутов материи.Однако энергия, импульс и угловой момент по-прежнему прочно занимают центральное место. Сохраняющаяся важность этих идей, унаследованных от классической механики, может помочь объяснить, почему этот предмет сохраняет такое большое значение в современной науке.

PHYS 200 — Лекция 9 — Вращения, Часть I: Динамика твердых тел

PHYS 200 — Лекция 9 — Вращения, часть I: Динамика твердых тел

Глава 1. Введение в твердые тела; Вращение твердых тел [00:00:00]

Профессор Рамамурти Шанкар: Хорошо, это новая тема под названием «Физика или динамика твердых тел».Итак, твердое тело — это то, что не сгибается, не меняет своей формы, когда вы прикладываете к нему силу, и одним из примеров может быть десятицентовик, монета или метр. Конечно, никто не является абсолютно жестким. Вы всегда можете согнуть что угодно, но мы возьмем приближение, которое есть у нас в руках, — абсолютно твердое тело. Техническое определение твердого тела состоит в том, что если вы выберете пару точек, расстояние между ними не изменится во время движения тела. Итак, динамика твердых тел в трех измерениях довольно сложна, потому что я хочу, чтобы вы сейчас представили себе не точечную массу, а некий объект некоторой формы, как этот парень.Вы берете это; вы подбрасываете его в воздух; вы можете видеть, что он делает что-то довольно сложное.

Фактически, мы можем охарактеризовать то, что он делает, с помощью двух выражений. Думаю, вы можете догадаться, что это такое. Во-первых, если бы это была точечная масса, то точечная масса просто двигалась бы туда-сюда или вверх-вниз. Он будет делать то, что называется «переводом». Он будет перемещаться из одного места в другое, может быть, по прямой, может быть, по кривой. Но на самом деле точечная масса не имеет дополнительной информации; ты должен сказать мне, где он и с какой скоростью движется.Вот и вся история. Но если у вас есть такое тело, этого недостаточно, чтобы сказать мне, где оно, потому что, если я подброшу его в воздух, вы можете представить, что когда оно движется по некоторой параболической траектории, в одно мгновение ластик может выглядеть так, а в следующий мгновенно и ластик могут выглядеть так; Итак, мы говорим, что он поступает и вращается. Это неочевидная теорема, что все, что тело делает от одного момента до следующего, может быть достигнуто, сначала переведя его, а затем сделав небольшое вращение, чтобы привести его к желаемой конфигурации.Другими словами, вы можете перейти отсюда сюда, выполнив перевод с последующим вращением.

Итак, перенос — это то, что мы постоянно изучаем, потому что для точечных частиц нет понятия вращения. Они берут точечную массу и вращают ее. Вы даже не подозреваете, что он вращается. У него нет внутренних переменных, поэтому мы и сосредоточили внимание на этой проблеме, но теперь мы собираемся расширить наше исследование на объекты, которые имеют размер и, следовательно, форму, которые имеют ориентацию.Недостаточно сказать, что футбол здесь. Мяч мог быть здесь, но его длинный заостренный конец мог быть сверху вниз или из стороны в сторону. Итак, мы хотим разбить проблему на две части. Мы хотим сосредоточиться на теле, которое только вращается, но не перемещается. Как только мы отточим свои навыки, мы вернем телу способность переводить.

Итак, сначала мы сосредоточимся на теле, которое не может быть переведено. Итак, это означает, что вы берете тело и захватываете в нем одну точку; тогда вы можете спросить: «Что он может сделать, если я поймаю в нем одно очко?» Что ж, любое движение будет таким, что точка не будет двигаться.Итак, в трех измерениях, если вы возьмете футбольный мяч и зафиксируете определенную точку, он может вращаться по-разному. Но в любой момент вы можете показать, что он будет делать. Он должен вращаться вокруг оси, проходя через точку, в которой вы его захватываете, потому что, если он поворачивается к оси, точки на оси не перемещаются, а точка, где вы его захватываете, лучше лежит на ось. Это тоже довольно сложно. Вы понимаете, почему я дал вам небольшую предосторожность. Трудно представить себе эти вещи.

Итак, я снова хочу рассмотреть простейшую возможную проблему; затем усложняйте его. Что ж, можно сказать, что самое простое, что вы делали весь семестр, — это рассматривать вещи в одном измерении. Но оказалось, что это слишком мало. Если вы находитесь в одном измерении и у вас есть твердое тело — кстати, твердое тело означает не только протяженное, но и твердое тело. Например, если вы возьмете кошку или змею, змею, которую вы можете схватить в одной точке, но змея не просто вращается вокруг одной точки, верно, пытается укусить вас, пытается двигаться; змея — это не твердое тело.Мертвая змея может быть твердым телом. Живая змея — определенно не твердое тело. Теперь вы должны помнить, что я дал вам передышку, изучая твердые тела, но настоящие тела на самом деле могут не только раскачиваться, скручиваться и поворачиваться, они также могут вибрировать и двигать руками. Это намного сложнее, поэтому мы не будем туда идти, а попробуем взять простейшее твердое тело. Но я сказал, что если я возьму твердое тело в одном измерении, оно вообще не может вращаться, потому что в одном измерении нет места для вращения.Он может двигаться только вперед и назад. Итак, чтобы показать вам, что есть какая-то новая возможность, я должен обратиться по крайней мере к двум измерениям.

Вот один пример, когда одного измерения недостаточно. Итак, я собираюсь представить твердые тела, которые живут в плоскости доски. Один из способов сделать это — взять кусок металла, просто вырезать форму, и это твердое тело. И, конечно, у этого есть небольшая толщина, потому что все, что вы делаете из материи, имеет некоторую толщину, по крайней мере, в один атом, но представьте, что эта толщина незначительна, поэтому это похоже на кусок металла, кусок фольги или что-то в этом роде, но оно жесткое, и это твердое тело, которое мы собираемся изучить.Это твердое тело может двигаться в плоскости доски и вращаться, но мы собираемся схватить его за одну точку. Итак, давайте выберем здесь какую-нибудь точку. Если я возьму его в этот момент и спрошу вас: «Что этот парень может сделать?» Итак, представьте, протыкаете его через доску и говорите: «Хорошо, делайте, что хотите». Вы понимаете, что все, что он может делать, — это вращаться вокруг оси, проникая в эту точку.

Итак, допустим, при t = 0 тело выглядело так, чуть позже оно будет выглядеть повернутым.Я должен сказать вам, что он делает в данный момент. Я должен сказать вам, как он ориентирован. Итак, для этого мы проводим через точку вращения некоторую линию. Вы можете выбрать любую понравившуюся строчку. Выберите такую ​​линию, и мы скажем, какой угол образует эта линия со стандартным выбранным направлением, обычно с осью x . Итак, если я дам вам θ , я думаю, вам придется подумать об этом и согласиться с тем, что я сказал вам, где находится тело. Вы можете реконструировать все, верно, потому что эта точка прибита; он не может двигаться.Вы берете тело; поверните его на угол θ .

Другими словами, вот вопрос. Предположим, вас нет в этой комнате; вы находитесь в другой комнате, и я хочу рассказать вам, что делает твердое тело. Какую информацию я должен сообщить вам, не видя картинки? Я утверждаю, что если я скажу вам, как линия была проведена в теле, что вы делаете раз и навсегда, линейчатая линия, то я скажу, что линейная линия образует угол θ с осью x .Полагаю, вы можете представить себе, находясь в другой комнате, что делает это тело, и в этом смысл слов: «Я предоставил полную информацию». Конечно, это не полная динамическая информация, потому что это также зависит от того, насколько быстро он вращается. Но, проще говоря, аналог того, где находится тело в одном измерении, — это угол θ . Итак, вы обнаружите, что мы собираемся провести аналогию между одномерным переносом и двухмерным вращением. Вы найдете эту аналогию очень полезной.Итак, я собираюсь использовать кусок доски, на котором я продолжу проводить аналогию. Я делаю это потому, что количество вещей, которые вам нужно запомнить, сократится, если вы математически сопоставите эту проблему с проблемой перевода в одном измерении.

Глава 2. Вращение через параметры окружности и радиан [00:08:15]

Другими словами, в одном измерении, когда тело движется, x говорит вам, где оно находится, верно? Я даю вам число, 3, или π , или -14; ты знаешь где это.Для этого твердого тела, живущего в плоскости доски, угол θ говорит вам, что это тело делает. Итак, даже если тело двухмерное, вам нужен один угол θ , чтобы определить его ориентацию. Вам не нужны два числа; одно число говорит вам, что делает твердое тело. Так что чуть позже его можно было повернуть на угол Δθ . Это аналог того, что он переместился с x на x + Δx .

Первое, что вам нужно сделать, это как вы хотите измерить θ ? Для x мы знаем, договорились.Мы собираемся измерять в метрах θ , стандартное предпочтение для людей с начальным статусом — измерять его в градусах. Вы измеряете его в градусах, и когда тело совершает один полный оборот, вы хотите сказать, что оно повернулось на угол в 360 градусов. Это не лучший способ измерения углов в более продвинутой динамике. Мы собираемся использовать что-нибудь еще. Вы можете себе представить, что это такое. Вы знаете, к чему я клоню. аКто-нибудь знает, куда я иду?

Студенты: радиан.

Профессор Рамамурти Шанкар: радиан. Я иду к радианам, и вы можете спросить себя, зачем кому-то думать о радианах? Что не так с 360 градусами? Я имею в виду, все учебники — я имею в виду, все романы говорят: «Когда я шел неправильным путем, я сделал 180». Верно? Итак, все знают. Фактически, я видел книгу скрипача, который сказал: «Я делал все не так, и я сделал 360». Итак, вы понимаете, что 360 градусов на самом деле не способ изменить вашу жизнь, но среди физиков он нашел отклик.С точки зрения физики, 360 градусов — действительно нетривиальная вещь. Оказывается, частицы в мире делятся на бозоны и фермионы, причем половина частиц — бозоны, а примерно половина — фермионы. Электрон — это фермион; квант света — это бозон, фотон. Для всех парней, которые являются фотонами или бозонами, если повернуть их на 360, они вернутся к тому, что делают. Оказывается, электрон, повернув его на 360, вычитает все числа, и только когда вы повернетесь на 720, вы вернетесь к тому, что технически делаете.Итак, вполне возможно, что этот человек имел в виду поведение; может быть, она была фермионом, но вот что было сказано: «Я сделал 360». Я проходил через 360 много раз, поэтому, если вы используете его, вам лучше подкрепить его той информацией, которую я дал, иначе вы просто ошибетесь в два раза. В любом случае, 360 — это очень хорошо. Я думаю, что вавилонянам понравился 360 по многим причинам. Делится на целую кучу чисел. Его легко разделить на более мелкие и мелкие части. Я считаю, что в некоторых инженерных кругах круг равен 400, а что-то еще, какая-то другая инженерная степень.Итак, вы можете решить, что такое круг, но нам нравится радиан.

Итак, давайте спросим: «Почему нам нравится радиан?» Для этого я хочу, чтобы вы взяли здесь точку на расстоянии r от центра и позволили ей переместиться в новое место под углом Δθ . Вы согласны с тем, что если твердое тело вращается, этот парень будет пересекать круг радиусом r . Поскольку это твердое тело, оно не может изменить расстояние от этой точки. Итак, мы движемся по кругу, и я спрашиваю вас, насколько велик этот сегмент, длина дуги ΔS , по которой он прошел, когда он повернулся на угол Δθ .Итак, один из способов вычислить это — посмотреть: если бы я сделал полный круг, я бы сделал 2 πR . r — расстояние этой точки твердого тела от центра. Я не прошел расстояние 2 πr , но я прошел Δθ , поэтому доля круга, который я охватил, составляет Δθ больше 360, если θ измеряется в градусах; Итак, позвольте мне указать это таким образом. Итак, мы можем записать π поверх 180 Δθ , умноженных на r .

Если вы измерили θ в градусах, это то, что вам нужно сделать, чтобы преобразовать угловой ход в фактическое расстояние по касательной к окружности.Линейное расстояние, которое вы путешествуете, связано с углом, на который вы поворачиваете, расстоянием r от центра и этим числом здесь. Поэтому люди говорят: «Послушайте, почему бы нам не облегчить жизнь, назвав θ умножить на π /180 в качестве нового угла». Назовем его Δθ умножить на r . Итак, Δθ, без какого-либо маленького нуля сверху, с этого момента означает радианы, и это связано с θ , поэтому, если вы проигнорируете Δ , само θ будет θ в градусах, /180.Тогда преимущество состоит в том, что расстояние, которое вы путешествуете — линейное расстояние, которое вы путешествуете — имеет простое отношение к углу, который вы путешествуете, длиной руки r от центра. Таким образом, вам не нужно носить с собой эти 180 и π и все такое, вы можете написать простой результат. Но теперь, когда вы это делаете, вы измеряете его в радианах, поэтому вы должны иметь представление о величине радиана. Ну, полный круг, если пройти полный круг; расстояние, которое вы пройдете, должно быть 2 πr , поэтому полный круг будет Δ θ = 2 π .Итак, круг стоит 2 π радиан; 2 π равно 6. Итак, один радиан примерно равен 1/6 от 360, что составляет 60 градусов.

Реальный ответ близок к 58 с чем-то градусам. Это выглядит странно. Странно, только если начать с 360 градусов. Но если вы встретили людей из чужой культуры, вам просто нужно было пойти в супермаркет, чтобы встретить этих людей, эти люди могут вообще не использовать 360; 360 — это человеческий артефакт, в 360 нет ничего особенного, независимого от нашей культуры или предубеждений.Это может быть связано с тем, сколько у нас пальцев. Это может быть связано с продолжительностью года и так далее. Но в числе 360 нет ничего естественного. С другой стороны, я считаю, что 2 π может быть обнаружено в любой развитой цивилизации. Если вы не вычислили π , значит, вы не развитая цивилизация. Если вы не догадались, 2, значит, вы сильно отстали. Итак, вы знаете 2, и вы знаете, что π , и это то, что будут использовать другие культуры, другие планеты и галактики.Это единица, которую мы любим использовать.

Вы снова и снова найдете людей, использующих естественные для нас единицы. Но позже, если вы захотите общаться с людьми на другой планете, вам придется использовать другую длину. Например, рост человека или длина руки Наполеона — это не очень хорошие единицы, потому что, если у вас нет Наполеона на вашем фоне, вы не понимаете, о чем говорите. Но если вы измеряете расстояния, насколько они велики, с точки зрения атома водорода, атомов водорода по всей Вселенной, это естественная шкала длины, которую можно использовать.Раньше это было очень популярно. Когда они запустили ракету или закопали что-то под землей, чтобы инопланетяне узнали, вы хотите показать, какой у нас рост; нет смысла давать ответ в метрах или футах или еще в чем-нибудь. Но если вы можете каким-то образом связать свой размер и размер атома водорода, тогда это число будет иметь смысл для людей во всех цивилизациях, потому что атомы водорода удобно производятся и разбросаны по всей вселенной, и они имеют одинаковый размер. Ладно, это длинная фраза о радианах, поэтому нам нравится использовать радианы.

Вы должны знать несколько популярных углов в радианах. Большую часть времени я буду использовать только радианы. Вы должны знать, что круг равен 2 π ; π — это полукруг, а π /2 — четверть круга, а когда вы находитесь под углом 90 градусов, это π /2; это то, что вам следует знать.

Как только вы напишете это так — давайте получим еще один очень полезный результат. Считайте расстояние, которое он проходит по касательным, за время, в течение которого он это делает.Это фактическая тангенциальная скорость или величина скорости в тангенциальном направлении. Я думаю, вы все согласны с этим; вот что мы с вами подразумеваем под скоростью. Если в какой-то момент этот кусок образца просто сломался и улетел, он продолжит лететь с такой скоростью. Эта штука, у нас есть обозначение ω и ω , называется угловой скоростью и измеряется в радианах в секунду.

Итак, вы можете рассчитывать угловую скорость для разных вещей. Например, какова угловая скорость Луны, когда она движется вокруг Земли? Ну, угловая скорость будет один оборот каждые 28 дней, верно? Один оборот равен 2 π радиан.В 28 дней вы делаете 28 раз 24 раза по 3600; это столько радианов в секунду. Или есть полотно бензопилы, и бензопила вращается с некоторым числом оборотов в минуту — может быть, 3000 оборотов в минуту — вы можете перевести число оборотов в минуту в число оборотов в секунду, но вы должны помнить, что каждый оборот составляет 2 π радиан. Итак, угловая скорость лезвия, вращающегося с частотой f , составляет 2 πf . Вы это понимаете? f — количество полных оборотов, которое он делает.Следовательно, если вы выберете точку на нем, он сделает ход 2 πf радиан. Итак, у словаря есть второй член, который представляет собой скорость, которая составляет dx / dt , теперь заменена угловой скоростью, которая составляет dθ / dt . Кстати, это аналогия. Это поможет вам сказать, что ω похоже на то, какая скорость была в старые времена, когда вещи двигались вперед и назад. Но теперь они крутятся по кругу, но даже здесь есть понятие скорости, но оно не в метрах в секунду, а измеряется в радианах в секунду.

Итак, вот кое-что, что вы, ребята, должны очень четко держать в голове. Я могу только упомянуть об этом вам, но я не могу сделать это за вас. Эта ω аналогична обычной скорости, но здесь также является действительной скоростью, старой доброй скоростью, то есть тангенциальной скоростью точек, которые являются частью вращающегося твердого тела. Это не входит в мой словарь, но это очень полезный результат. Тангенциальная скорость — это угловая скорость, умноженная на расстояние от этой точки до центра вращения.Таким образом, твердое тело в целом имеет одну угловую скорость. Вот что значит быть жестким. Все вращается. Вы не можете сказать, что угловая скорость такая-то здесь и так-то там; это собственность всего тела. С линейной скоростью это не везде одинаково; точки рядом здесь в то же время идут так далеко, а точки рядом здесь в то же время идут так далеко. Следовательно, линейная скорость увеличивается от центра. Угловая скорость постоянна, и твердое тело не может иметь двух угловых скоростей.Он может иметь только одну угловую скорость.

Глава 3. Радиальное и тангенциальное вращение при постоянном ускорении [00:19:57]

Хорошо, тогда мы говорим: «Хорошо, давайте возьмем задачу, в которой угловая скорость сама по себе изменяется. Хорошо, это аналог того, что есть скорость изменения скорости, и здесь у нас есть величина, называемая α , которая является скоростью изменения угловой скорости или второй скоростью изменения угла, и это обозначается по α .Итак, если вы взяли одну из этих вращающихся дисковых пил, и она вращалась с определенной скоростью, и из-за трения, скажем, она замедлялась, или когда вы включаете двигатель, она ускоряется, предположим, вы только что повернули вниз двигатель пилы, и он ускоряется во вращении, вы должны четко понимать, что ускорение деталей — это пильное полотно, хорошо? Я пытаюсь нарисовать для вас одну из этих пил, а она крутится. Если вы выберете точку, скажем, на окружности; Как вы помните, давно мы узнали, что даже если он движется с постоянной скоростью, с постоянной угловой скоростью, он имеет тангенциальную скорость, которая составляет ω умножить на r .И из-за этого у него будет ускорение к центру, центростремительное ускорение, которое составляет v 2 / r , и вы можете записать его как ω 2 r . Это единственное ускорение, которое у него есть, даже если он вращается с постоянной скоростью. Но я говорю, что у него может быть и тангенциальное ускорение, если, помимо всего прочего, изменяется сама угловая скорость.

Что ж, давайте не торопимся, чтобы это переварить. Это не так уж сложно, но я хочу, чтобы вы поняли.Если он вращается с устойчивым углом ω , мы узнали, что все, что движется по кругу, имеет старомодное линейное ускорение к центру, которое составляет v 2 / r . Но мы поняли, что v в тангенциальном направлении составляет ωr , поэтому, если вы поместите это — если хотите, можете назвать это v тангенциальным — это дает мне ω 2 r , это просто v 2 / р . Но если вы притормозите его, увеличив или уменьшив скорость, у него будет, вдобавок, ускорение в этом направлении.Итак, ускорение точки на краю или где-либо еще может иметь две составляющие. Тангенциальный будет присутствовать только в том случае, если вы ускоряете или замедляете вращающийся диск, а радиальный всегда будет там, пока он вращается.

Хорошо, когда у вас есть этот словарь, у вас может быть целая куча аналогичных величин. Например, если вы сосредоточитесь на задачах, в которых тело имеет постоянный угол ускорения α , что вы можете сказать? Что ж, вы уже можете сказать, что θ будет θ 0 + ω 0 + ½ αt 2 .Это потому, что θ ведет себя так же, как x . Если бы x имело постоянное ускорение a , мы все знали, что x было равно начальному x + начальная скорость, умноженная на время, плюс ½ при 2 . Точно так же угол, на который он поворачивается, будет начальным углом. Начальная угловая скорость умножена на т плюс что-то, что включает т 2 . Это только в том случае, если α — константа, точно так же, как я надеюсь, вы, ребята, знаете, что это верно, только если a — константа.Хорошо, теперь я собираюсь провести аналогию и спросить, какова кинетическая энергия твердого тела, когда оно вращается? Да?

Студент: Не могли бы вы объяснить [неразборчиво]

Профессор Рамамурти Шанкар: Здесь?

Студент: Нет, ускорение с α .

Профессор Рамамурти Шанкар: Здесь?

Студент: Рядом с пильным полотном.

Профессор Рамамурти Шанкар: А здесь?

Студент: Up.

Профессор Рамамурти Шанкар: Этот?

Студент: Ага.

Профессор Рамамурти Шанкар: Хорошо, я говорю, если у вас есть вращающийся диск, возьмите любую точку на вращающемся диске, даже если диск движется с постоянной угловой скоростью, он имеет тангенциальную скорость и направление вращения. скорость постоянно меняется. Это самое старое, что мы узнали, поэтому у него есть ускорение к центру. Но это не угловое ускорение; это просто стандартное ускорение к центру всего, что движется по кругу.Это всегда будет. Но вдобавок ко всему, если вы увеличите или уменьшите скорость вращения диска, у вас также будет тангенциальное ускорение, которое составляет α умножить на r . Еще не понятно, или? Простые люди знают об этом ускорении. Если вы находитесь на вращающейся платформе и кто-то увеличил скорость вращения, вы заметите, что вы двигаетесь быстрее. Это ускорение в тангенциальном направлении. Ускорение к центру — более сложная задача, которую мы узнали, и это связано с тем, что скорость — это вектор, а не скаляр.Итак, если направление меняется, есть ускорение, которое мы ассоциируем с этим, и оно так же реально, как и любое другое ускорение; ему нужна сила, чтобы это произошло.

Может быть, лучший способ спросить вас вот что. Предположим, это маленькое пятнышко и есть ты, хорошо? Вы случайно застряли на вращающемся полотне бензопилы и цепляетесь за него изо всех сил. Какую силу вам нужно приложить, чтобы оставаться на вращающемся лезвии? Возможно, с вами и со мной такого не случится, но если вы являетесь участником организованной преступности, это не очень необычная ситуация.Вы можете столкнуться с различными механизмами и хотите знать, что я делаю, чтобы остаться. Что ж, Ньютон подскажет вам ответ. Ньютон говорит, что если вы хотите быть на этом вращающемся полотне бензопилы, ваше ускорение к центру должно быть оплачено, и если лезвие набирает скорость, за это тоже нужно заплатить. Это чистое ускорение; это, раз ваша масса будет силой, которая должна быть приложена к вам. Итак, вы будете цепляться за него с этой силой, и эта сила, в свою очередь, будет приложена диском к вам по третьему закону.

Например, предположим, что это вид сверху на вращающуюся платформу. Вы просто стоите, а платформа вращается. Вы можете спросить: «Какая сила трения мне нужна, чтобы оставаться на платформе?» Что ж, вам нужна сила, чтобы предоставить вам ma , и я говорю вам, что ваш a состоит из двух частей. Даже если карусель вращается с постоянной скоростью, вам понадобится сила трения, направленная к центру, чтобы согнуть вас в круг. Теперь, если наверху, карусель начинает увеличивать или уменьшать скорость.Нам также понадобится тангенциальная составляющая силы, чтобы обеспечить тангенциальное ускорение.

Хорошо, теперь это кинематика вращения при постоянном угловом ускорении, которая математически идентична этой, потому что математически, если вы знаете d 2 θ / dt 2 — это α , ответ такой же, как d 2 x / dt 2 = a . Вы меняете символы, получаете тот же ответ. Итак, вы можете представить себе множество проблем, которыми я не хочу заниматься.Я думаю, вы легко справитесь с этим. Я бы предпочел сосредоточиться на более сложных.

Например, полотно бензопилы, вращающееся со скоростью 3600 об / мин, замедляется с некоторой скоростью α . Сколько времени нужно, чтобы отдохнуть? Итак, для этого вы напишете вторую формулу: ω = ω 0 + αt . Это аналог v = v 0 + на . И вы начинаете с какого-то ω 0 , вы нажимаете на тормоза; α это отрицательное число, вы видите, в какое время это ω переходит в 0; вот когда это прекратится.Другой результат, который я буду использовать сегодня, фактически следующий: ω 2 = ω 0 2 + 2α (θ — θ 0 ) . То есть аналог v 2 = v 0 2 + 2 a (x — x 0 ) . Это говорит о том, что если тело имеет ускорение α , то оно, конечно, будет набирать скорость, а конечный квадрат скорости связан с квадратом начальной угловой скорости плюс удвоенное ускорение, умноженное на угол поворота.Итак, это аналогия, поэтому нам не нужно изобретать все эти вещи заново. Они просто возникают из-за того, что математически одинаковые вещи имеют одинаковые математические результаты. Они подчиняются одному и тому же уравнению; у них одинаковые результаты.

Глава 4. Момент инерции, угловой момент, кинетическая энергия [00:28:34]

Хорошо, теперь давайте найдем кинетическую энергию твердого тела. У него масса, и если он вращается, все маленькие атомы, составляющие тело, движутся, и у них есть свои собственные ½ mv 2 .И мы хотим спросить: «Какова сумма ½ mv 2 по всем частицам?» Для этого удобно взять следующее более простое твердое тело. Возьмем твердое тело, взяв массу м 1 на расстоянии r 1 , соединенное с какой-то центральной ступицей безмассовым стержнем, возьмем другую массу и берем третью массу. Это м 2 , на расстоянии r 2 , там м 3 , на расстоянии r 3 и так далее.Вы можете создать твердое тело, смоделировав его таким простым способом. Если вы немного более абстрактны, вы можете взять непрерывное твердое тело и проделать то же самое; но давайте начнем с простого. Вот твердое тело, три жестких стержня с массами м 1 , м 2 , м 3 , и вот он прибит, а там шпажка, проходящая через доску, а вещи могут только повернуть вокруг этого.

Кстати, я должен сказать вам соглашение для ω .В одномерном движении скорость положительна, вы движетесь вправо, а при отрицательной — влево. С угловой скоростью у нас должно быть соглашение о том, что положительно, а что отрицательно. Эта вещь, которая крутится, может вращаться только по часовой стрелке или против часовой стрелки. Оказывается, ω положительна для против часовой стрелки. Просто люди используют такое соглашение. Фактически, вы заметили, как я рисовал твердое тело, когда я рисовал Δθ , у меня было вращение против часовой стрелки.Это предпочтительное направление для измерения угла θ от оси x .

Хорошо, возьмем это твердое тело. Он вращается с общей угловой скоростью. Надеюсь, вы все понимаете, что значит быть жестким. Итак, у всех одинаковая угловая скорость ω . Итак, какова кинетическая энергия этого объекта? Ну, это кинетическая энергия каждой массы, суммированная по всем массам i , m i v i 2 .Теперь, какова скорость каждого объекта? Если взять это тело, м 1 , его скорость обязательно перпендикулярна линии, соединяющей его с точкой вращения. Этот движется сюда; он не может двигаться по соединяющей его линии, потому что в противном случае это не твердое тело. Для твердого тела все расстояния сохраняются. Итак, вы не подходите близко к центру и не уходите далеко от центра. Итак, все, что вы можете сделать, это повернуть вокруг центра. Итак, скорость этого парня v 1 = ωr 1 .Скорость этого оттяжки v 2 , такая же ωr 2 и так далее. Итак, вы можете все это сложить. А что вы получите? Вы получаете ½ суммы по i m i r i 2 ω 2 . И вы должны произвести суммирование по всем телам. В моем примере суммирование идет от 1 до 3. У вас может быть 1 до бесконечности и 1 до миллиона; это не имеет значения. Это кинетическая энергия.

Ребята, вы следите за этим? Это не новая кинетическая энергия; это то, что мы называем кинетической энергией, всего ½ мВ 2 для каждой части, составляющей твердое тело.Для удобства я решил, что твердое тело состоит из небольшого набора масс. Но теперь, если вы заметили, при переходе от 1 к 3 м 1 и м 2 и м 3 — это три разных числа. Мы не знаем массы. r 1 и r 2 и r 3 — три разных числа. Мы не знаем; каждый из них может находиться на разном расстоянии от начала координат.Но у ω нет нижнего индекса i ; нет ничего, что называется ω для этого и ω для этого. На все есть только одна ω . Итак, вы можете вытащить это из суммы и написать так. Затем ответ мы запишем как идентичный ½ 2 , и дадим этому I имя; I называется моментом инерции. Итак, продолжая аналогию, эта плата будет использоваться для аналогии, у нас было K = ½ mv 2 .Тем не менее, мы можем иметь K = ½ 2 и I называется моментом инерции.

Итак, момент инерции определяется не только массами, составляющими тело, но и тем, насколько они удалены от центра. Если бы все массы просто упали на вершину центра, у тела не было бы момента инерции. Он будет весить одинаково; момент инерции исчезнет. Точно так же, если масса разложена, момент инерции больше. Например, если я стою здесь, а вы подходите и решаете крутить меня, я стою вот так, вы пытаетесь крутить меня, я высовываю руки, мой момент инерции изменился, потому что сумма моя масса теперь находится дальше от оси вращения.Итак, момент инерции зависит от расположения масс. Мы берем твердое тело; это не похоже на танцоров на льду. Мы говорим о твердом теле; следовательно, момент инерции постоянен. Но для определения момента инерции требуется расчет.

И еще один важный момент. Если я решу повернуть тело в новом месте, скажем, вокруг этой точки, момент инерции изменится, потому что изменятся все расстояния r . Момент инерции — если кто-то скажет: «Вот массы, вот где они, пожалуйста, найдите мне момент инерции», — вы скажете: «Я не могу этого сделать.Вы скажете: «Я не могу этого сделать, пока вы не скажете мне точку, вокруг которой вы планируете вращать тело. Если это не дано, я не могу сказать вам, каков момент инерции ». Следовательно, момент инерции относительно точки, и нет ничего, что называется массой относительно точки. Масса — это просто масса. Момент инерции зависит от точки, вокруг которой вы вычисляете момент инерции; это переменная. Также стоит знать, что если тело вращается с массой м 3 , то м 3 не влияет на момент инерции; это из.Все, что находится на оси, не влияет. Вещи, которые находятся дальше, вносят больший вклад пропорционально своей массе, пропорционально квадрату расстояния от оси.

Итак, это мой словарь. Итак, кто играет роль массы? Масса играется моментом инерции. В нем есть килограммы, квадратные метры, и никто не дал им названия. Например, ньютон-умножитель называется джоуль. Вы можете спросить, ну, это килограмм на метр в квадрате, может быть, он назван в честь кого-то.Пока он не назван в честь кого-либо, он просто называется Ньютон-метр в квадрате. Итак, это будет аналог массы в нашем мире вращений. Вы помните, в мире вращения [поправка: следовало бы сказать «перевод»] мы определяем то, что называется импульсом, который равен массе, умноженной на скорость. Здесь мы будем называть то, что называется угловым моментом, будет I умножить на ω . Этот L называется угловым моментом.

Один чрезвычайно важный момент, который вы, ребята, должны заметить, заключается в том, что все используемые мной концепции, такие как масса и энергия, являются теми же старыми концепциями, которые мы узнали ранее.Тот факт, что это вращающееся тело, ничего не меняет, понятно? Кинетическая энергия по-прежнему означает то же самое; этот K , хотя и выглядит по-другому, на самом деле представляет собой ½ mv 2 каждой части тела, суммированной по телам. Итак, этот L называется угловым моментом. Чтобы привыкнуть к этому, нужно время, но сейчас это произведение момента инерции и угла скорости. Итак, теперь мы ищем самое важное уравнение в механике, которое составляет F = ma .Это будет похоже на какой-то загадочный объект. Теперь вы должны уметь угадывать, что происходит с правой стороны, хорошо? К настоящему времени вы должны знать, по аналогии, что это может быть записано как dL / dt , если хотите, скорость изменения количества движения, или это может быть записано как I умножить на α ; это не имеет значения.

Я надеюсь, вы понимаете, что если вы возьмете скорость изменения импульса, вы получите ma , потому что, когда вы берете производную, m не меняется.Точно так же здесь, если вы возьмете dL / dt , I не меняется, ω изменяется, давая вам dω / dt , что составляет α . I не меняется, потому что, когда тело вращается, конечно, частицы в теле движутся, но их расстояние от точки вращения не меняется. Несмотря на то, что тела перемещаются в новые места, r 2 остается прежним, потому что они просто вращаются. Вот почему в производной по времени вам не нужно брать производную по времени от момента инерции.

Но мы не знаем, кто будет играть роль слева, понятно? Итак, мы это найдем. Мы собираемся найти аналог этого. Итак, я собираюсь сказать вам, что есть много способов узнать это. Это гораздо более тонкий вопрос, чем принято считать, о том, каков правильный закон. Вы можете слышать, как часто говорится о законе, но некоторые из его аргументов неверны. Итак, позвольте мне рассказать вам один способ добраться до закона. Надеюсь, вы знаете, на что я смотрю. Я пытаюсь выяснить, какая величина возможна для скорости изменения углового момента, или каков аналог левой части для мА .Я знаю, что правая часть — это . По аналогии, кто слева? Во-первых, совершенно очевидно, что если вы хотите, чтобы момент импульса тела изменился, необходимо изменить ω . Вы можете увидеть, вращается ли что-то при фиксированном ω , вы хотите это изменить, вам нужно толкать, тянуть или что-то делать. Итак, очевидно, что вы собираетесь применить силу, но то, что появляется слева, — это не просто сила, а что-то еще, и мы выясним, что это такое.И это делается с помощью следующего устройства.

Я собираюсь использовать старую добрую теорему о рабочей энергии, которая гласит: когда сила действует на тело, работа, которую оно совершает, которая равна силе, умноженной на расстояние, которое оно проходит, равна изменению кинетической энергии. Это стандартная теорема о рабочей энергии, которую мы установили. Итак, если вы взяли такое твердое тело, и давайте применим к твердому телу, я просто возьму более простой случай твердого тела с этими двумя массами, m 2 и m 1 ; это не имеет значения.Теперь я прилагаю силу F , перпендикулярную этому телу, на расстоянии r от точки вращения. И пусть это вращает тело в это время; скажем, тело перемещается на величину Δθ . Может быть, мне стоит нарисовать вам для этого картинку получше, потому что это очень, очень важно. Итак, вот тело; Я просто взял самый простой, с двумя массами, м 1 , м 2 . Я просто собираюсь подтолкнуть его сюда, и я решаю подтолкнуть его в этом направлении, и он находится на расстоянии r от точки вращения.И пусть он повернется на небольшой угол и придет в это положение, а этот угол равен Δθ . Какую работу выполняет эта сила? Я собираюсь это записать, но хочу, чтобы вы об этом подумали. Работа, совершаемая силой, — это сила, умноженная на расстояние, на которое она прошла. И в основном, я спрашиваю вас, на какое расстояние он переместился, и если вы хоть что-нибудь помните из того, что я делал ранее, то примерно эта длина дуги, которую мы видели, составляет r умножить на Δθ .

Другими словами, если вы приложите силу F и повернете тело на Δθ , расстояние, на которое вы фактически переместитесь, составит r Δθ ; Итак, это сила, умноженная на расстояние. Теперь это должно быть равно изменению кинетической энергии. Оно должно быть равно изменению кинетической энергии. Напомню, что кинетическая энергия ½ 2 . Следовательно, изменение кинетической энергии будет ½ I (ω 2 — ω 0 2 ) .Кстати, это два очень соседних момента времени. Если в соседние моменты времени, в течение этого короткого периода мы можем считать, что тело имеет постоянное ускорение; следовательно, α постоянно. Таким образом, по этой формуле вы также можете рассчитать, когда α является постоянным, период. В реальном теле, когда оно вращается, α не обязательно должно быть постоянным, но для достаточно небольшого интервала, сколь угодно малого, мы можем применить формулу, которая работает, когда α является константой.В этом случае вы запишете это как ½ умноженное на 2 α, углов поворота. Верно? Посмотрите на эту формулу здесь. Я имею в виду этот результат: ω, , окончательный квадрат, -ω, , начальный квадрат, в 2 раза больше, чем α, раз больше, чем изменение угла. Итак, я хочу, чтобы вы приравняли эту проделанную работу к изменению кинетической энергии. Я хочу, чтобы вы приравняли этого парня к этому парню, и мы получили отличный результат: F умножить на r = I умножить на α . составляет мА ; вот что я ищу.Итак, этот тип, Fr , является аналогом силы, и он называется крутящим моментом.

Очень важно заметить, что крутящий момент, создаваемый этой силой, равен значению силы, умноженному на расстояние от точки вращения, и я говорю только о внешней силе. В стержне также действуют внутренние силы. Каждую часть стержня тянет за собой другая часть стержня, но этот анализ с точки зрения сил можно провести, но это очень сложно и сложно.Намного легче смотреть на энергию, в которой энергия изменяется только под действием внешних сил. Если вы смотрите только на силы, вы должны быть осторожны. Эта точка поворота также прикладывает силу к штанге, но она не движется и не выполняет никакой работы. Итак, единственная внешняя сила — это та, которую я прилагаю, и она отвечает за изменение кинетической энергии. Внутри стержня есть другие силы. Другими словами, эта масса будет набирать скорость не потому, что я толкаю ее напрямую. Я, вероятно, толкаю эту часть стержня, а она толкает часть рядом с собой, и наконец она толкает эту.Итак, я не применяю закон силы напрямую к этой массе. Я говорю: что бы ни происходило внутри, меня не волнует; умножение внешней силы на расстояние — это изменение кинетической энергии объекта. Да?

Студент: Но как вы говорите угловой [неразборчиво]

Профессор Рамамурти Шанкар: Нет, я сказал, что эта величина, изменение кинетической энергии, является конечной кинетической энергией, умноженной на ½ ω 2 минус начальная кинетическая энергия; но окончательный / начальный относится к тому моменту, когда стержень был там, и стержень повернулся на небольшую величину.

Студент: Справа.

Профессор Рамамурти Шанкар: Хорошо.

Студент: Итак, вы говорите, что это 2 α из [неразборчиво]

Профессор Рамамурти Шанкар: Справа.

Студент: Вы сказали, что нужно что-то сказать о скорости [неразборчиво]

Профессор Рамамурти Шанкар: Здесь. Нет, я использовал это уравнение, изменение, ω 2 в конце минус ω 2 в начале, это 2-кратное ускорение и угол ускорения, умноженный на θ θ — это не изменение θ за это время, верно.Другими словами, эта формула может использоваться даже для конечных периодов времени при условии, что α постоянна. Мы применяем его в течение крошечного периода времени, достаточно малого, чтобы мы могли поверить, что ускорение за этот крошечный период составляет какое-то число α . Итак, это ω 2 — ω 0 2 — это то, что действительно, если вы хотите, чтобы изменение ω 2 было пропорционально α , умноженному на изменение угла. Итак, α не является константой; α может изменяться от момента к моменту.Но в этот момент, в зависимости от сил, будет примерно α , а то, что называется крутящим моментом, составляет I умноженное на α . Теперь, я думаю, очень ясно, что если бы на тело действовало несколько сил, то крутящий момент был бы F i r i , и есть только одно последнее предостережение по этому поводу, одна финальная квалификация, а именно: что на практике силы, которые используются для вращения тела, не всегда могут быть перпендикулярными. Вы можете применить силу в другом направлении, например.

Глава 5. Теорема о крутящем моменте и рабочей энергии [00:46:47]

Теперь вы должны понять, что крутящий момент — это способность изменять вращательное состояние тела, и мы понимаем, что он зависит от силы, которую вы прикладываете, и от того, насколько далеко сила от точки вращение. Итак, если у вас есть дверь, вы пытаетесь открыть дверь, вот дверь, вот петли, и вы хотите открыть эту дверь, применив силу. Возникает вопрос: «Куда поставить дверную ручку?» Хорошо? Вы вдруг придумали дверь, вы подумали о петлях, но так и не поняли, куда поставить дверную ручку.Вы спросите себя: «Ну, у меня полная свобода, может быть, мне будет хорошо, если я поставлю его прямо рядом с петлями», тогда вы поймете, что есть части вселенной, такие как Земля Моронов, где, если вы пойдете, все петли, все дверные ручки здесь, и они говорят: «Вы знаете, мы прикладываем много силы, мы никуда не денемся. Что не так?» Что ж, мы понимаем, что в то время как сила — это все в линейном движении, сила — это не все во вращательном движении. Если вы хотите окупить свои деньги, вам нужно взять дверную ручку как можно дальше и поставить ее ближе к концу.

Хорошо, тогда вы разберетесь с этим. Затем вы подходите и говорите: «Хорошо, я поставил дверную ручку в нужном месте, откройте дверь», и вы прилагаете таким образом огромную силу. И снова разочарованный, снова вы не получаете результатов, вы говорите: «Я ухожу как можно дальше от петли», и вы понимаете, что если вы действительно хотите, чтобы что-то пошло, вам действительно следует принять меры. линия разделения между тем местом, где он вращается, и местом, где вы прикладываете силу, и приложите силу, перпендикулярную ей.Любая часть, параллельная ему, ничего не делает, если только вы не оторвете ее от петель. Ты не хочешь этого делать. Итак, у этой силы есть бесполезная часть и полезная часть, а бесполезная часть, которая пытается оторвать стержень от петель, будет уравновешена силой от петель, потому что стержень представляет собой жесткий стержень, он не сдвинется с места. Но тот, который перпендикулярен стержню, на самом деле может повернуть стержень, не влияя на жесткость.

Итак, нам нужен дополнительный множитель, который равен sin θ i , где θ i определяется следующим образом.Если вы находитесь в точке r i и таким образом прикладываете силу F , то θ — это угол между линией, соединяющей точку приложения силы, и направлением силы. Вы находитесь здесь, и вы разделены этим вектором, и как угол между этим вектором разделения и силой, этот угол θ может изменяться от силы к силе. Но эта сила F i , угол может быть θ i . Это окончательная формула крутящего момента, хорошо? Итак, мы собрали все части, и это окончательный ответ.Это определение крутящего момента, и с помощью этого крутящего момента мы теперь можем дополнить словарь и сказать то, что мы хотим. Слева находится крутящий момент, где крутящий момент определяется как F i r i sin θ i . F sin θ — это просто составляющая силы, перпендикулярная разделению r . Вот что делает F sin θ . Надеюсь, вы это видите. Если у вас есть сила, действующая таким образом, F cos θ является этой частью, что не годится, а F sin θ — той частью, которая подходит для вращения.Кроме того, если вы хотите, с точки зрения проделанной работы, вам нужна сила, умноженная на пройденное расстояние. Пройденное расстояние только в этом направлении. В этом направлении нет расстояния. Итак, пройденное расстояние составляет F скалярное произведение с направлением, в котором вы двигаетесь, и это относится только к силе, перпендикулярной разделению, а не к параллельной части.

В любом случае, я надеюсь, мы интуитивно понимаем, почему именно этот парень отвечает за вращение объектов. Вы хотите повернуть что-то вокруг какой-либо оси, у вас есть определенное количество силы, вам лучше всего приложить силу как можно дальше от точки вращения.Другими словами, предположим, что мы хотим, чтобы задний конец этого стола оставался неподвижным, но я хочу повернуть его так, чтобы это было так. Тогда я говорю, я думаю, мы все знаем, что мы хотим начать здесь продвигаться. Кроме того, мы не хотим прикладывать силу тем или иным образом, мы хотим приложить силу перпендикулярно. Если по какой-то причине мы привязаны к веревке и можем тянуть только в этом направлении, то составляющая силы таким образом должна быть исключена, бесполезно, это та часть, которая делает поворот, и эта часть также делает работу.Никакая сила, перпендикулярная движению, не будет делать никакой работы. Никакая сила, перпендикулярная перемещению, не будет работать. Это твердое тело может быть только позже в этом положении, поэтому смещение является тангенциальным направлением, и только сила в тангенциальном направлении вносит вклад в работу. И в моем аргументе об энергии работы это то, что здесь происходит. Это причина, по которой вы пришли к такому определению крутящего момента.

Хорошо, у нас есть большая часть деталей для этого. Еще одно напишу по аналогии.Работа, совершаемая силой, перемещающей тело на величину dx , такова. Работа, совершаемая крутящим моментом, составит τ раз — думаю, вы можете догадаться, что я хочу туда поставить — раз или Δθ . Когда крутящий момент вращает тело на угол Δθ , то проделанная работа будет равна τ умножить на Δθ . Вы можете видеть, что даже здесь вы можете записать выполненную работу как F умножить на r умножить на Δθ ; но это крутящий момент, это Δθ .Так что это очень полезно знать. Итак, если вы, ребята, собираетесь рассуждать по аналогии, я бы попросил вас узнать аналоги, только тогда вы сможете решать проблемы. Итак, давайте здесь привыкнем к этой формуле: ΔW = τ Δθ . Я покажу вам один пример.

Предположим, у меня есть стержень, свисающий с потолка. Ну, допустим, он безмассовый, внизу масса м и какая-то длина L ; это маятник. Итак, я возьму этот маятник и приведу его в это положение под углом θ 0 , который, к сожалению, θ 0 обозначает конечный угол.Я хочу знать, сколько работы я делаю; вот что мы собираемся вычислить. Итак, мы здесь занимаем промежуточную позицию. Какая сила действует на этого парня? Этот стержень безмассовый; усилие на нем составляет мг . Если этот угол равен θ , вы можете видеть, что угол равен θ , поэтому mg cos θ действует таким образом, а mg sin θ действует таким же образом. Чтобы удержать маятник от падения при угле θ , я должен приложить тангенциальную силу в этом направлении размером мг sin θ .Надеюсь, вы это видите. мг cos θ будет обеспечиваться стержнем. Поскольку стержень — твердое тело, он не позволяет массе двигаться в этом направлении. Он обеспечит любую силу, необходимую для радиального движения массы. В тангенциальном направлении, чтобы он не соскользнул обратно туда, где он начался, мне нужно приложить силу мг sin θ . Итак, крутящий момент, который я прилагаю, составляет мг sin θ , умноженное на L . Итак, я сделал все это, взяв силу, обнаружив компонент силы, перпендикулярный объекту, и уравновесил его противодействующей силой.Вы также можете сказать, что крутящий момент, который я должен приложить, действительно составляет мг , что представляет собой силу, R , которая здесь составляет L , и синус угла между ними. Как бы мы это ни писали, это крутящий момент.

Давайте посмотрим, как работает этот крутящий момент, когда я поворачиваю его от начального угла до некоторого конечного угла. Это будет mgL sin θ dθ от 0 до некоторого конечного угла θ 0 . Я просто говорю, что проделанная работа — это τ Δθ, просуммированная по всем углам; это означает интегрировать.Итак, вам нужно вернуться к своему исчислению, и вы должны помнить, что интеграл sin θ равен — cos θ , и вы хотите начать с θ = 0 до θ , равного некоторый конечный угол θ 0 . А если вы проворачиваете его, вы обнаружите, что это мг / л, умножить на 1 — cos θ . Итак, вот работа, которую мы проделали, чтобы взять маятник, который находится в горизонтальном, вертикальном положении, и повернуть его на угол θ 0 . Я собираюсь провести перекрестную проверку, чтобы сказать вам, что это работает с другой точки зрения.Эта масса была здесь, теперь она поднялась туда. Итак, вы можете спросить: «Что случилось с моей работой?»

Возвращаясь к теореме о рабочей энергии, возьмем здесь эту массу. На это действовали силы. Один из них был стержнем, но сила стержня всегда была перпендикулярна движению этого боба, поэтому он не мог выполнять никакой работы. Единственная работа была сделана мной. Что случилось с моей работой? Работа, которую я проделал, должна была изменить общую энергию тела, но боб не двигался до этого и не двигался после, потому что я даю только правильную силу, чтобы нейтрализовать гравитацию; Я не придавал ему никакой скорости.Итак, должно быть, потенциальная энергия этого боба, когда он здесь, должна объяснить работу, которую я проделал. Это именно то, что есть, потому что позвольте мне переписать это как [ mg ] — mgL + mgL cos θ . Но, может быть, мы увидим, что это мг + мг. Этот, ну, я думаю, лучше всего записать его как L — L cos θ , и вы, ребята, сможете увидеть, что происходит. L — это расстояние здесь, L cos θ — это расстояние, а L — L cos θ — это увеличение высоты этого боба.Эта высота здесь на самом деле составляет L — L cos θ , потому что, если это L и это θ , это будет L cos θ до этой точки. Итак, это всего лишь мг, умножить на ч , и вот где ваша работа.

Итак, в чем смысл этого упражнения? Это упражнение поможет вам привыкнуть к вычислению проделанной работы не как сила, умноженная на расстояние, а как крутящий момент, умноженный на угол. И вот проблема, когда сам крутящий момент изменяется с углом.Крутящий момент сам по себе является функцией θ , поэтому вам нужно сделать интеграл, и вот как вы получите работу. Итак, теперь у нас есть все необходимое оборудование, все механизмы, необходимые для выполнения кинематики или динамики твердых тел в этой аналогии.

Итак, позвольте мне резюмировать, что это такое. У вас твердое тело; у него есть момент инерции; у него угловая скорость. Произведение двух называется угловым моментом, L , и чтобы его изменить, вам нужно приложить крутящий момент.Чтобы найти крутящий момент, найдите все внешние силы, действующие на тело, умножьте каждую силу на расстояние от точки вращения, но составляющая каждой силы, перпендикулярная линии разделения, умноженная на расстояние разделения, это Fr sin θ и сложите его. Опять же, когда вы затягиваете крутящий момент, вы должны подать знак. Если крутящий момент имеет тенденцию вращать тело против часовой стрелки, мы будем считать его положительным, а если крутящий момент стремится вращать тело по часовой стрелке, мы будем считать его отрицательным.Итак, возможно, что в данном теле могут быть моменты, пытающиеся сдвинуть его в разных направлениях. Так, например, в простейшем твердом теле, состоящем, скажем, из двух масс, вы можете толкать в одну сторону, а я — в другую. Мы все пытаемся вращать их в противоположных направлениях. Приложенный крутящий момент будет равен вашей силе, умноженной на это расстояние; это называется положительным, потому что вы движетесь против часовой стрелки. Крутящий момент, который я прилагаю, будет равен моей силе, умноженной на меньшее расстояние, и это будет считаться отрицательным, потому что он направлен по часовой стрелке.И вы их сложите; вы получите чистый крутящий момент. Этот чистый крутящий момент вызовет ускорение этого похожего на гантель объекта.

Какое ускорение это вызовет? Это будет моя сила, умноженная на мое расстояние, sin θ оказывается равным 1, минус ваша сила, умноженное на расстояние, на котором вы это делаете, умноженное на момент инерции. Момент инерции для простой задачи равен m 1 r 1 2 + m 2 r 2 2 , если это m 1 и это r 1 , а это м 2 и это расстояние составляет r 2 .Итак, это простейшая задача, которую вы могли бы решить в динамике твердого тела. Возьмите пару сил, вычислите крутящие моменты, разделите их на момент инерции и получите угол ускорения. Если крутящий момент постоянный, его трудно поддерживать. Что мне нужно сделать, чтобы поддерживать постоянный крутящий момент? Я должен начать бегать с этим, пока он вращается, и вы можете себе представить, как это сделать; тогда крутящий момент будет постоянным, αa будет постоянным, α постоянным будет как постоянная , и вы сможете решить всю кучу задач, которые вы, ребята, делали раньше.Хорошо, теперь есть одно техническое препятствие, которое вам нужно преодолеть, если вы хотите заниматься динамикой твердого тела, и это знать, как вычислить момент инерции для всех типов объектов. Если я дам вам 37 гирь, каждая на расстоянии х от точки вращения, это тривиальная вещь. Сделайте mr 2 для каждого и добавьте его. Но вот такие объекты, которые вам могут быть даны, не дискретный набор масс, а сплошной шарик. Как и в случае с центром масс, когда у вас есть группа масс, вы взяли m 1 x 1 + m 2 x 2 + m 3 x 3 и т. Д. на.Но если у вас есть стержень, который является непрерывным, вам нужно сделать интеграл. Таким образом, вам также придется сделать интеграл здесь.

Глава 6. Расчет момента инерции: примеры для стержня, диска и т. Д. [01:01:36]

Итак, давайте возьмем некоторые твердые тела и попытаемся найти момент инерции, которые являются непрерывными твердыми телами. Итак, вот кольцо радиуса r и массы M . Это не диск, несмотря на то, как он выглядит. Это кольцо, и я хочу найти его момент инерции.Итак, вы должны подумать о том, что вы будете делать. Итак, вы думаете на секунду, и я скажу вам, как я это сделаю, — это разделить его на крошечные части, каждая из которых достаточно мала, чтобы я мог обдумать точку. Этот крошечный кусок имеет массу Δm i , а i идет от 1, 2, 3, 4 до любого, 10 миллионов, пока я не обойду круг. Момент инерции будет — я использую Δm и , чтобы сказать вам, что это крошечная масса для этой секции. Но что такое r 2 ? Если я планирую повернуть его вокруг этой точки, я планирую повернуть вокруг этой точки; это же r 2 для всех.Итак, вы можете вытащить это из суммирования и получить просто Mr 2 . Итак, это очень простая проблема. Это так же просто, как и эта проблема, но у вас явно была только одна масса, и вся она находилась на расстоянии r от точки вращения, это Mr 2 . Здесь масса распределена, но, к счастью, она распределена таким образом, что каждая ее часть находится на одинаковом расстоянии r от центра, поэтому очень легко произвести суммирование. Вытяните r 2 , просуммируйте массу и получите общую массу.Тогда мы займемся… Есть вопросы по этому поводу? Все будет построено на понимании этого. Итак, для кольца это момент инерции, который равен Mr 2 .

Но теперь я хочу сделать следующее. Хочу сейчас взять диск, радиусом r ; Мне нужно вычислить сумму Mr 2 для каждой массы в задаче. Итак, вот где вам нужно организовать свое мышление. Планирую сделать момент инерции через центр. Во-первых, если бы я не сказал вам момент инерции, через который вы даже не можете начать; Я говорю вам, что хочу через центр.Чтобы протолкнуть его через центр, мы можем представить его как состоящий из целой группы концентрических колец, каждое с радиусом r и толщиной dr . Если вы можете найти момент инерции этой крошечной заштрихованной области, я суммирую все маленькие шайбы, кольца, которые у меня есть, чтобы заполнить весь диск. Итак, каков момент инерции этого кольца? Что ж, я тебе уже сказал. Если кольцевое пространство имеет массу M , тогда это Mr 2 — момент инерции этого кольцевого пространства.Итак, вопрос: «Какова масса заштрихованной области?» Очевидно, что r 2 , который мы используем, — это r , на котором он сидит. Но какую массу мне следует использовать? Что ж, для массы, которую я должен использовать, я утверждаю, что если общая масса равна [M], то умноженное на [должно быть сказано «больше»] πR 2 — это масса на единицу площади. Затем мне нужна область этой заштрихованной области. Это сложная часть. Это у тебя есть? Да?

Студент: 2 πrdr .

Профессор Рамамурти Шанкар: Верно, давайте спросим, ​​почему это 2 πrdr . Если вы возьмете это кольцо, возьмете ножницы, вырежете его и откроете, оно будет выглядеть как длинный прямоугольник, толщина которого будет dr ; его длина составит 2 πr . Итак, 2 πrdr — это то, что говорит вклад, вот dr . Теперь вам нужно сложить это по всему. Сложение — это то, что мы делаем, выполняя интеграл.Я не хочу делать этот интеграл; вы, ребята, можете хотя бы догадываться, что произойдет. В ответе не будет π ; π больше нет. У вас есть rr 2 is r 3 , r 4 /4, и если вы объедините это с этим, вы получите MR 2 /2. Итак, момент инерции диска MR 2 /2. Это потому, что если бы я пришел и сказал вам момент… предположим, я где-то сделал ошибку и сказал, что это MR 2 , знаете ли вы, что я ошибаюсь? И почему вы будете утверждать, что это не может быть правильный ответ? Да?

Студент: Ну, это как кольцо, только радиусы меньше.

Профессор Рамамурти Шанкар: Справа. Другими словами, его ответ, правильный ответ: если это MR 2 , это означает, что вся масса находится на расстоянии R 2 . Но мы знаем, что часть массы намного ближе к центру. Фактически, некоторые из них находятся прямо в центре, поэтому вы не можете получить тот же R 2 в качестве вклада от всех частей. Некоторые будут на нулевом расстоянии; некоторые будут на полной дистанции. Итак, всякий раз, когда вы определяете момент инерции для диска, он должен выглядеть как MR 2 , умноженное на число, которое определенно меньше единицы.Оказывается, половина; если бы у нас был третий или четвертый, вы бы не знали, что это явно неправильно. Но если я получил 1 впереди или того хуже, если я получил 2 раза MR 2 , то вы знаете, что я совершил ошибку.

Хорошо, теперь последний из объектов, на которые я хочу сегодня взглянуть, — это стержень, который я хочу взять. На самом деле это тот пример, который довольно часто используется в качестве первого примера, но я сделаю это сейчас. Итак, вот жезл, хорошо? У него длина L , масса M , и я спрашиваю, какой момент инерции? Вы знаете, что вы могли сделать? Какой первый вопрос вы зададите, когда я скажу: «Каков момент инерции?

Студент: Примерно в какой точке?

Профессор Рамамурти Шанкар: Примерно в какой точке, верно? Пока я не скажу тебе этого, ты не сможешь начать.Итак, я скажу: «Давайте найдем момент инерции вокруг этого конца». Итак, что вы должны представить в этом случае, вы воткнете сюда гвоздь, и парень будет вращаться вот так по большому кругу с центром в центре. Если вы воткнете гвоздь в эту середину, это другая история, и ответ будет другим. Итак, вот что я делаю, я беру расстояние x . Беру ленту толщиной dx . Эта полоска достаточно мала, чтобы я мог подумать, что это точечная масса. Следовательно, момент инерции этой крошечной части будет выглядеть как квадрат расстояния, умноженный на массу крошечной части.Масса крошечной порции выглядит так. Дайте-ка подумать; это масса на единицу длины, это длина, это масса объекта. Затем мне нужно сделать интеграл от 0 до L . Ну, x 2 даст мне x 3 /3. Это даст мне ML 2 /3, и это момент инерции стержня. Опять же, он выглядит как ML 2 , перед ним должно стоять число меньше 1, потому что, если вы измеряете отсюда, самое большое расстояние, которое вы можете пройти, это L ; многим парням было намного ближе.Таким образом, средневзвешенное значение квадрата расстояния не может быть всего L 2 ; это должно быть что-то меньшее, чем L 2 . Тот факт, что это 1/3 — это, конечно, то, ради чего вы делаете всю работу.

Студент: Вам также понадобится L .

Профессор Рамамурти Шанкар: Простите меня?

Студент: An L тоже.

Профессор Рамамурти Шанкар: Вы забыли L здесь?

Студент: Ага.

Профессор Рамамурти Шанкар: Кроме того, измерение момента инерции должно быть в квадрате конечной массы, умноженной на длину. Да? Тебя это не устраивает?

Студент: Нет, не знаю.

Профессор Рамамурти Шанкар: Вы согласны, хорошо. Послушайте, иногда я делаю это неправильно, так что вам всегда нужно следить за этим. Последнее, что я хочу сделать, это найти… Ну, когда я говорю последнее, что хочу сделать, я не имею в виду, что я не хочу этого делать, я просто говорю, что близок к концу день.Последнее, что я сейчас вычислю, — это момент инерции вокруг центра. Посмотрим, что будет дальше. Это тот же самый интеграл, x 2 , dx / L , M / L , но диапазон интеграции для x теперь изменится с -L /2 до + L /2. . Затем я упрощаю свою жизнь, записывая это как 2 M / L умножить на x 2 dx от 0 до L /2. Я постоянно призываю вас к этому.Если интеграл является четной функцией x , а именно, -x дает столько же, сколько + x , тогда вы можете взять половину причины интеграции и удвоить ответ. Вы можете сделать это, только если это четная функция, так что если x и — да?

Студент: Почему он все еще превышает L , если вы поместите x вместе с другим?

Профессор Рамамурти Шанкар: Ой, извините, вы абсолютно правы. Способ сделать это здесь.Видишь, ты продолжаешь попытки, ты поймаешь меня. Правильно, правильно. Хорошо, давайте сделаем это сейчас, 2 M / L , хорошо, теперь я действительно волнуюсь, потому что это L 3 , это X 3 /3, но то, что строится в кубе это L /2, и это дает мне ML 2 /12. Итак, момент инерции — это не фиксированное число. Когда я повернул его вокруг центра масс, он стал намного меньше. Когда я взял его за левый край, у меня получилось ML 2 /3; Дошел до мидпойнта, получил ML 2 /12.Думаю, совершенно очевидно, что я собираюсь сократить ответ. Так что, когда я буду продолжать, он станет еще меньше. Вот где ваш здравый смысл скажет вам, что если вы получите какой-то ответ оттуда, вы получите тот же ответ отсюда, и вы достигнете наилучшего возможного момента инерции. Вы должны уметь аргументировать интуитивно, и если вы произведете расчет, то окажетесь так же.

Теперь, что нужно проверить, если вы это сделаете, момент инерции вокруг одного конца — это момент инерции вокруг центра за вычетом массы, умноженной на ( L /2) 2 .Это то, что вы действительно можете проверить. Просто возьмите числа, которые я вам дал, и вы обнаружите, что этот ответ плюс M раз ( L /2) 2 будет равен этому ответу. И это очень общая теорема, о которой я расскажу в следующий раз. Если вы знаете момент инерции через центр масс, все готово, хорошо? Если мне нужен момент инерции через любую другую точку, я беру момент инерции через центр масс и добавляю к нему общую массу объекта и квадрат расстояния, на которое вы заставили меня переместить мою ось, что здесь оказывается L /2.Я, конечно, докажу это; Я не хочу ничего вам давать без доказательств. На самом деле в нашем классе мы можем убедиться, что это правда. Это верно не только для стержня, но и для диска. Это верно для любого объекта в двух измерениях: если у вас есть момент инерции этого существа через центр масс и вы хотите, чтобы он проходил через эту точку, просто возьмите ответ через центр масс и прибавьте к нему Md 2 где d — это расстояние.Это будет доказано в следующий раз. Хорошо, вы знаете, что это неделя, когда мы встречаемся в понедельник, среду и пятницу.

[конец стенограммы]

Наверх Третий закон

Ньютона для вращательного движения. Динамика материальной точки и поступательное движение твердого тела. Материальная точка и твердый

1. Производная по времени импульса K материальной точки или системы материальных точек относительно фиксированной (инерциальной) системы отсчета равна главному вектору F всех внешних сил, приложенных к системе:
dK / dt = F или mac = F

, где ac — ускорение центра инерции системы, а m — ее масса.
В случае поступательного движения твердого тела с абсолютной скоростью v скорость центра инерции vc = v. Следовательно, при рассмотрении поступательного движения твердого тела это тело можно мысленно заменить материальным точка, которая совпадает с центром инерции тела, которое имеет всю свою массу и движется под действием главного действующего лица внешних сил, приложенных к телу.
В проекциях на ось неподвижной прямоугольной декартовой системы координат уравнения основного закона динамики поступательного движения системы имеют вид:
Fx = dK / dt, Fy = dK / dt, Fz \ u003d dK / dt

или
macx = Fx, macy = Fy, macz = Fz

2.Простейшие случаи поступательного движения твердого тела.
а) Движение по инерции (F = 0):
mv = const, a = 0.

b) Движение под действием постоянной силы:
d / dt (mv) = F = const, mv = Ft + mv0,

где mv0 — величина движения тела в начальный момент времени t = 0.
в) Движение под действием переменной силы. Изменение величины движения тела за интервал времени от t1 до t2 составляет
mv2 — mv1 = Fcp (t2 — t1)

, где Fcp — среднее значение вектора силы на временном интервале от t1 до t2.

Прочие записи

10.06.2016. Первый закон Ньютона

1. Первый закон Ньютона: каждая материальная точка поддерживает состояние покоя или равномерного и прямого движения до тех пор, пока удар других тел не выведет ее из этого состояния. Это …

10.06.2016. Мощность

1. Сила — это векторная величина, которая является мерой механического воздействия на материальную точку или тело других тел или полей. Сила полностью указана, если ее числовое значение, направление…

10.06.2016. Третий закон Ньютона

1. Действия двух материальных точек друг на друга численно равны и направлены в противоположные стороны: Fij = — Fji, где i не равно j. Эти силы действуют в разных точках и могут уравновешиваться …

Поступательное движение — это механическое движение системы точек (тела), при котором любой отрезок прямой линии, связанный с движущимся телом, форма и размеры которого не изменяются во время движения, остается параллельным его положению в любом месте. предыдущий момент времени.Если тело движется поступательно, то для описания его движения достаточно описать движение произвольной его точки (например, движение центра масс тела).

Одной из наиболее важных характеристик движения точки является ее траектория, которая в общем случае представляет собой пространственную кривую, которую можно представить в виде сопряженных дуг разного радиуса, каждая из которых исходит из своего центра, положение которых может меняться время. В пределе прямую можно рассматривать как дугу, радиус которой равен бесконечности.

В этом случае получается, что при поступательном движении в каждый данный момент времени любая точка тела совершает оборот вокруг своего мгновенного центра вращения, и длина радиуса в данный момент одинакова для всех точки тела. Векторы скорости точек тела, а также испытываемые ими ускорения одинаковы по величине и направлению.

Например, кабина лифта движется вперед. Также в первом приближении кабина колеса обозрения совершает поступательное движение.Однако, строго говоря, движение кабины колеса обозрения нельзя считать поступательным.

Основное уравнение динамики поступательного движения произвольной системы тел

Скорость изменения количества движения системы равна главному вектору всех внешних сил, действующих на эту систему.

Второй закон Ньютона — основной закон динамики поступательного движения — отвечает на вопрос о том, как изменяется механическое движение материальной точки (тела) под действием приложенных к ней сил.Учитывая действие различных сил на данную материальную точку (тело), ​​ускорение, приобретаемое телом, всегда прямо пропорционально равнодействующей данных приложенных сил:

При действии одной и той же силы на тела с разной массой ускорения тел оказываются разными, а именно

С учетом (1) и (2), а также того факта, что сила и ускорение являются векторными величинами, можно записать

Соотношение (3) и есть второй закон Ньютона: ускорение приобретаемая материальной точкой (телом), пропорциональная вызывающей ее силе, совпадает с ней по направлению и обратно пропорциональна массе материальной точки (тела).В системе измерения СИ коэффициент пропорциональности k = 1. Тогда

Учитывая, что масса материальной точки (тела) в классической механике постоянна, в выражение (4) массу можно ввести под знаком производной:

Векторная величина

, численно равная произведению массы материальной точки на ее скорость и имеющая направление скорости, называется импульсом (величиной движения) этой материальной точки. Подставляя (6) в (5), получаем

Это выражение является более общей формулировкой второго закона Ньютона: скорость изменения количества движения материальной точки равна силе, действующей на нее.

Основные характеристики поступательного движения:

1.path — любое движение по траектории

2. Перемещение — кратчайший путь.

А также сила, импульс, масса, скорость, ускорение и т.д.

Число степеней свободы — это минимальное количество координат (параметров), установка которых полностью определяет положение физической системы в пространстве.

При поступательном движении все точки тела в каждый момент времени имеют одинаковую скорость и ускорение.

Закон сохранения момента импульса (закон сохранения момента количества движения) является одним из фундаментальных законов сохранения. Математически он выражается через векторную сумму всех угловых моментов вокруг выбранной оси для замкнутой системы тел и остается постоянным до тех пор, пока на систему не будут действовать внешние силы. Соответственно, угловой момент замкнутой системы в любой системе координат не меняется со временем.

Закон сохранения момента количества движения — это проявление изотропии пространства по отношению к вращению.Это следствие второго и третьего законов Ньютона.

Экспериментальные исследования взаимодействий различных тел — от планет и звезд до атомов и элементарных частиц — показали, что в любой системе взаимодействующих тел при отсутствии действия сил от других тел, не входящих в систему, или равенстве суммы действующих сил к нулю, геометрическая сумма импульсов тел остается неизменной.

Система тел, которые не взаимодействуют с другими телами, не входящими в эту систему, называется закрытой системой.

P-Pulse

(с векторами)

14. Различия между вращательным и поступательным движением. Вращательная кинематика … Вращательное движение — это тип механического движения. При вращательном движении абсолютно твердого тела его точки описывают окружности, расположенные в параллельных плоскостях. Поступательное движение — это механическое движение системы точек (тела), при котором любой отрезок прямой, связанный с движущимся телом, форма и размеры которого не меняются во время движения, остается параллельным его положению в любой предыдущий момент. вовремя .[ Существует близкая и далеко идущая аналогия между движением твердого тела вокруг фиксированной оси и движением отдельной материальной точки (или поступательным движением тела). Каждой линейной величине из кинематики точки соответствует аналогичная величина из кинематики вращения твердого тела. Угол φ соответствует координате s, угловая скорость w соответствует линейной скорости v, а угловое ускорение ε соответствует линейному (тангенциальному) ускорению a.Сравнительные параметры движения:

Поступательное движение

Вращательное движение

Move S

Угловое смещение φ

Линейная скорость

Угловая скорость

Разгон

Угловое ускорение

Момент инерции I

Момент импульса

Момент силы M

Рабочий:

Рабочий:

Кинетическая энергия

Кинетическая энергия

Закон сохранения импульса (MMP)

Закон сохранения углового момента (ZSMI)

При описании вращательного движения твердого тела относительно неподвижного тела в заданной системе отсчета обычно используются векторные величины специального вида.В отличие от рассмотренных выше полярных векторов r (радиус-вектор), v (скорость), a (ускорение), направление которых естественным образом следует из природы самих величин, направление векторов, характеризующих вращательное движение, совпадает с направлением оси вращения, поэтому их называют осевыми (лат. axis — ось).

Элементарный поворот dφ — это осевой вектор, модуль которого равен углу поворота dφ, а направление вдоль оси вращения OO »(см. Рис.1.4) определяется правилом правого винта. (Угол поворота твердого тела).

Рисунок 1.4. Для определения направления осевого вектора

Линейное перемещение dr произвольной точки A твердого тела связано с радиус-вектором r и поворотом dφ соотношением dr = rsinα dφ или в векторной форме через векторное произведение:

dr = (1.9)

Соотношение (1.9) справедливо именно для бесконечно малого вращения dφ.

Угловая скорость ω — это осевой вектор, определяемый производной по времени вектора вращения:

Вектор ω, как и вектор dφ, направлен вдоль оси вращения согласно правилу правого винта (рис.1.5).

Рисунок 1.5. Для определения направления вектора

Угловое ускорение β — это осевой вектор, определяемый производной по времени вектора угловой скорости:

β = dω / dt = d2φ / dt2 = ω «= φ» «

При ускоренном движении вектор β по направлению совпадает с ω (рис. 1.6, а), а при медленном движении векторы β и ω направлены противоположно друг другу (рис. 1.6, б).

Фигура 1.6. Связь направлений векторов ω и β

Важное замечание: решение всех задач о вращении твердого тела вокруг фиксированной оси по форме аналогично задачам о прямолинейном движении точки. Достаточно заменить линейные величины x, vx, ax соответствующими угловыми величинами φ, ω и β, и мы получим уравнения, аналогичные (1.6) — (1.8).

Период обращения

(Время, за которое тело совершает один оборот)

Частота (количество оборотов в единицу времени) —

Динамика — это раздел механики, изучающий движение материальных тел вместе с физическими причинами, вызывающими это движение.

Динамика основана на законах Ньютона.

1. Закон инерции … Существуют такие CRM, в которых любое тело может находиться в состоянии покоя или равномерного прямолинейного движения до тех пор, пока действие сил упругости не изменит его состояние.

Этот закон рассматривает тело как материальную точку и выполняется только в ISO.

Сила — это физическая величина, которая характеризует воздействие на данное тело со стороны других тел, вызывая изменение движения тела.

2. Закон движения материальной точки. Тело импульс … Скорость изменения количества движения материальной точки равна действующей на нее силе F:

Изменение количества движения в момент времени dt равно равнодействующим силам.

3. Право взаимодействия. Если одно тело воздействует на другое с некоторой силой, то второе тело действует на первое с той же силой.

Эти силы всегда имеют одну и ту же природу, равны по величине, противоположны по направлению и приложены к различным телам.

Динамика материальных точек. Основные уравнения движения материальной точки в дифференциальной форме.

Динамика системы частиц, центр инерции системы, закон движения центра инерции.

Рассмотрим систему точек с массами m1, m2… m n.

Центр масс — точка, для которой выполняется радиус-вектор:

Центр масс изолированной системы покоится или находится в равномерном прямолинейном движении.

Закон движения центра масс — в инерциальных системах отсчета центр масс системы движется как материальная точка, в которой находится масса всей системы, и на которую действует сила, равная к геометрической сумме всех внешних сил, действующих на систему.

Динамика системы частиц, закон сохранения количества движения в замкнутой системе.

В отсутствие сил импульс материальной точки остается неизменным по величине и направлению (следствие второго закона Ньютона).

Перепишем его для системы из N частиц:

, где суммирование ведется по всем силам, действующим на n-ю частицу с m-й стороны. Согласно третьему закону Ньютона силы формы и будут равны по модулю и противоположны по направлению, то есть, тогда после подстановки результата в выражение (1) правая часть будет равна нулю, то есть:

или

Как известно, если производная некоторого выражения равна нулю, то это выражение является константой относительно переменной дифференцирования, что означает:

(постоянный вектор).

То есть полный импульс системы частиц является постоянной величиной.

Поворот тела на некоторый угол можно задать как отрезок длиной j, направление которого совпадает с осью, вокруг которой выполняется вращение. Направление вращения и представляющий его сегмент связаны правилом правого винта.

В математике показано, что очень маленькие повороты можно рассматривать как векторы, обозначенные символами или.Направление вектора вращения связано с направлением вращения тела; — вектор элементарного вращения тела — псевдовектор, так как не имеет точки приложения.

При вращательном движении твердого тела каждая точка движется по окружности, центр которой лежит на общей оси вращения (рис. 6). В этом случае радиус-вектор R , направленный от оси вращения к точке, поворачивается во времени Dt на некоторый угол Dj … Угловая скорость и угловое ускорение вводятся для характеристики вращательного движения.


Угловая скорость называется векторной величиной, равной первой производной угла поворота тела по времени:

Угол в 1 радиан — это центральный угол, длина дуги которого равна радиусу круг; 360 о = 2р рад.

Направление угловой скорости задается правилом правого винта : вектор угловой скорости сонаправлен с вектором, то есть с поступательным движением винта, головка которого вращается в направлении движения точки по оси длина окружности.

Линейная скорость точки связана с угловой скоростью:

В векторном виде.

Если угловая скорость изменяется во время вращения, возникает угловое ускорение.

Угловое ускорение Векторная величина, равная первой производной угловой скорости по времени. Вектор угловой скорости сонаправлен с вектором элементарного изменения угловой скорости, произошедшего за время dt:

.

При ускоренном движении вектор параллельный (рис.7), при медленном — наоборот (рис. 8).

Угловое ускорение возникает в системе только при изменении угловой скорости, то есть при изменении линейной скорости движения по величине. Изменение скорости по величине характеризует тангенциальное ускорение.

Найдем связь между угловым и тангенциальным ускорениями:

.

Изменение направления скорости при криволинейном движении характеризуется нормальным ускорением:

.

Таким образом, связь между линейными и угловыми величинами выражается следующими формулами:

Типы вращательного движения:

а) переменная — движение, в котором меняют а:

б) равнопеременный — вращательное движение с постоянным угловым ускорением:

дюйма) равномерное — вращательное движение с постоянной угловой скоростью:

.

Равномерное вращательное движение можно охарактеризовать периодом и скоростью.

Период — это время, за которое тело совершает один полный оборот.

Частота вращения Число оборотов, сделанных за единицу времени.

За один оборот :,

, г.

Законы Ньютона. Основное уравнение динамики поступательного движения.

Dynamics изучает движение тел с учетом причин, вызывающих это движение.

Динамика основана на законах Ньютона.

I закон. Существуют инерциальные системы отсчета (IFR), в которых материальная точка (тело) поддерживает состояние покоя или равномерного прямолинейного движения до тех пор, пока действие других тел не выведет его из этого состояния.

Свойство тела сохранять состояние покоя или равномерного прямолинейного движения при отсутствии воздействия на него других тел называется инерцией .

IFR называется системой отсчета, в которой тело, свободное от внешних воздействий, покоится или движется равномерно по прямой линии.

Инерциальный — это система отсчета, которая находится в состоянии покоя или движется равномерно прямолинейно относительно любой ППП.

Система отсчета, движущаяся с ускорением относительно IFR, не является инерциальной.

Закон Ньютона I, также называемый законом инерции, был впервые сформулирован Галилеем. Его содержание сводится к 2 утверждениям:

1) все тела обладают свойством инерции;

2) ISO существует.

Принцип относительности Галилея : все механические явления во всех ИСО происходят одинаково, т.е.е. никакими механическими экспериментами внутри ИСО невозможно установить, находится ли данная ИФ в покое или движется равномерно по прямой линии.

В большинстве практических задач система отсчета, жестко связанная с Землей, может рассматриваться как ИСО.

Из опыта известно, что при одних и тех же ударах разные тела по-разному изменяют свою скорость, т.е. приобретают разное ускорение, причем ускорение тел зависит от их массы.

Вес — мера инерционных и гравитационных свойств тела.Путем точных экспериментов было установлено, что инертная и гравитационная массы пропорциональны друг другу. Выбирая единицы таким образом, что коэффициент пропорциональности становится равным единице, мы получаем, что, следовательно, они просто говорят о массе тела.

[м] = 1кг — масса платино-иридиевого цилиндра, диаметр и высота которого равны h = d = 39мм.

Для характеристики действия одного тела на другое вводится понятие силы.

Мощность — мера взаимодействия тел, в результате которой тела изменяют свою скорость или деформируются.

Прочность характеризуется числовым значением, направлением, точкой приложения. Прямая линия, вдоль которой действует сила, называется линией действия силы .

Одновременное действие нескольких сил на тело эквивалентно действию одной силы, называемой равнодействующей или результирующей силой и равной их геометрической сумме:

Второй закон Ньютона — основной закон динамики поступательного движения — отвечает на вопрос о том, как изменяется движение тела под действием приложенных к нему сил.

Раздел механики, изучающий движение материальных тел вместе с физическими причинами, вызывающими это движение, называется динамикой. Основные концепции и количественные закономерности динамики возникли и развиваются на основе многовекового человеческого опыта: наблюдений за движением земных и небесных тел, производственной практики и специально поставленных экспериментов.

Великий итальянский физик Галилео Галилей экспериментально установил, что материальная точка (тело) достаточно далеко от всех других тел (т.е., не взаимодействуя с ними) будет сохранять состояние покоя или равномерного прямолинейного движения. Эта позиция Галилея подтверждалась всеми последующими экспериментами и составляет содержание первого основного закона динамики, так называемого закона инерции. В этом случае отдых следует рассматривать как частный случай равномерного и прямолинейного движения, когда.

Этот закон одинаково справедлив как для движения гигантских небесных тел, так и для движения мельчайших частиц. Свойство материальных тел поддерживать состояние равномерного и прямолинейного движения называется инерцией.

Равномерное и прямолинейное движение тела при отсутствии внешних воздействий называется движением по инерции.

Система отсчета, относительно которой выполняется закон инерции, называется инерциальной системой отсчета. Инерциальная система отсчета почти в точности гелиоцентрическая. Ввиду огромного расстояния до звезд их движением можно пренебречь, и тогда оси координат, направленные от Солнца к трем звездам, не лежащим в одной плоскости, будут неподвижны.Очевидно, что любая другая система отсчета, движущаяся равномерно и прямолинейно относительно гелиоцентрической системы координат, также будет инерциальной.

Физической величиной, характеризующей инерцию материального тела, является его масса. Ньютон определил массу как количество вещества, содержащегося в теле. Это определение нельзя считать исчерпывающим. Масса характеризует не только инерцию материального тела, но и его гравитационные свойства: сила притяжения, испытываемая данным телом со стороны другого тела, пропорциональна их массам.Масса определяет общий запас энергии материального тела.

Понятие массы позволяет уточнить определение материальной точки. Материальная точка — это тело, при изучении движения которого можно абстрагироваться от всех его свойств, кроме массы. Следовательно, каждая материальная точка характеризуется величиной своей массы. В механике Ньютона, основанной на законах Ньютона, масса тела не зависит от положения тела в пространстве, его скорости, действия других тел на тело и т. Д.Масса — это аддитивная величина, т.е. масса тела равна сумме масс всех его частей. Однако свойство аддитивности теряется при скоростях, близких к скорости света в вакууме, т.е. в релятивистской механике.

Эйнштейн показал, что масса движущегося тела зависит от скорости

, (2.1)

где m 0 — масса тела в покое,  — скорость тела, с — скорость света в вакууме.

Из (2.1) следует, что при движении тел с малой скоростью c масса тела равна массе покоя, т.е.е. м = м 0; при c масса равна m.

Обобщая результаты экспериментов Галилея по падению тяжелых тел, астрономические законы Кеплера о движении планет, данные собственного исследования, Ньютон сформулировал второй основной закон динамики, который количественно связал изменение движения планет. материальное тело с силами, вызывающими это изменение в движении. Остановимся на анализе этого важнейшего понятия.

В общем, сила — это физическая величина, характеризующая действие одного тела на другое.Эта векторная величина определяется числовой величиной или модулем
, направлением в пространстве и точкой приложения.

Если на материальную точку и действуют две силы, то их действие эквивалентно действию одной силы

,

, полученной из известного треугольника сил (рисунок 2.1). Если на тело действуют n-силы, общее действие эквивалентно действию одной равнодействующей, которая является геометрической суммой сил:

. (2.2)

Динамическое проявление силы состоит в том, что под действием силы материальное тело испытывает ускорение.Статическое действие силы приводит к тому, что упругие тела (пружины) под действием сил деформируются, газы сжимаются.

Под действием сил движение перестает быть равномерным и прямолинейным и появляется ускорение (), его направление совпадает с направлением действия силы. Опыт показывает, что ускорение, получаемое телом под действием силы, обратно пропорционально величине

его массы:

или
.(2.3)

Уравнение (2.3) представляет собой математическую запись второго основного закона динамики:

вектор силы, действующей на материальную точку, численно равен произведению массы точки на вектор возникающего ускорения. от действия этой силы.

Начиная с ускорения

,

, где
— единичные векторы,
— проекция ускорения на оси координат, затем

. (2.4)

Если обозначено, то выражение (2.4) можно переписать в терминах проекции сил на оси координат:

В системе СИ единицей силы является ньютон.

Согласно (2.3) ньютон — это такая сила, которая сообщает ускорение в 1 м / с 2 массе в 1 кг. Нетрудно заметить, что

.

Второй закон Ньютона можно записать иначе, если ввести понятие количества движения тела (m) и импульса силы (Fdt). Подставляем в

(2.3) выражение для ускорения

,

, получаем

или
.(2.5)

Таким образом, элементарный импульс силы, действующий на материальную точку за промежуток времени dt, равен изменению количества движения тела за тот же промежуток времени.

Обозначив импульс тела

,

, мы получим следующее выражение для второго закона Ньютона:

.

В релятивистской механике при c основной закон динамики и количества движения тела с учетом зависимости массы от скорости (2.1.) Запишется в следующем виде:

,

.

До сих пор мы рассматривали только одну сторону взаимодействия между телами: влияние других тел на характер движения данного выбранного тела (материальной точки). Такое влияние не может быть односторонним, взаимодействие должно быть взаимным. Этот факт находит отражение в третьем законе динамики, сформулированном для случая взаимодействия двух материальных точек: если материальная точка m 2 опыта со стороны материальной точки м 1 сила равна, тогда м 1 испытывает м 2 сила, равная по величине и противоположная по направлению:

.

Эти силы всегда действуют вдоль прямой линии, проходящей через точки м 1 и м 2 , , как показано на рисунке 2.2. Рисунок 2.2, и относится к

Динамика вращательного движения. Момент инерции. Теорема Штейнера. Кинетическая энергия вращающегося тела. Момент силы. Динамическое уравнение вращательного движения. Угловой момент. Закон сохранения момента количества движения.

Динамика вращательного движения.
§ 1 Момент инерции. Теорема Штейнера


Момент инерции точки
равен

Момент инерции системы относительно оси вращения называется физической величиной, равной сумме произведения масс n материальных точек на квадраты их расстояний от оси. из.

Момент инерции тела при непрерывном распределении массы

-интегрирован повсюду.

1. Находим момент инерции однородного диска относительно оси, перпендикулярной плоскости диска и проходящей через его центр. Диск разделяем на кольцевые слои толщиной d r . Все точки слоя будут на одинаковом расстоянии от оси равном r . Объем такого слоя

Кольцо квадратное

2.Стенный полый цилиндр радиуса R (обруч, велосипедное колесо и т.п.).

3. Цельный цилиндр или диск с радиусом R

4. Ось прямого тонкого длинного стержня перпендикулярна стержню и проходит через его середину.

5. Шар радиуса R вокруг оси, проходящей через центр

Если вам известен момент инерции относительно оси, проходящей через ее центр масс, момент инерции относительно любой оси, параллельной этой, определяется с помощью теоремы Штейнера: момент инерции относительно оси I вращение параллельно моменту инерции C относительно параллельно оси, проходящей через центр масс C тела, со сложенным куском тела массой m и квадратом расстояния между осями

6.Момент инерции прямого стержня длиной, ось которого перпендикулярна стержню и проходит через его конец.

§ 2 Кинетическая энергия вращения


Рассмотрим твердое тело, вращающееся вокруг неподвижной оси Z , проходящее через нее с угловой скоростью ϖ. поскольку тело абсолютно жесткое, поэтому все тело будет вращаться с одинаковой угловой скоростью

Если разбить тело в малых количествах до элементарных масс м 1 , м 2 … на расстоянии r 1 , r 2 …, от оси вращения кинетическую энергию тела можно записать как

Известно, что или тогда

Из сравнения W K rot с W k поступательного движения видно, что момент инерции вращательного движения заменяет массу во вращательном движении и является мерой инерции тела.

Если тело совершает поступательные и вращательные движения одновременно, это

Например, цилиндр катится без проскальзывания по плоскости.

§ 3 Момент.

Динамическое уравнение вращательного движения твердого тела


Момент сила (крутящий момент) относительно фиксированной точки O называется значением псевдовектора, равным векторному произведению радиус-вектора от точки O до точки приложения силы, сила .

-псевдовектор его направление совпадает с плоскостью движения правого винта при его повороте от до. Направление крутящий момент также можно определить по правилу левой руки, четыре пальца левой руки положить в направлении первого фактора, второй фактор находится в ладони, согнутой под прямым углом к большой палец указывает направление крутящего момента. Момент вектора силы всегда перпендикулярен плоскости, в которой векторы и

— где кратчайшее расстояние между линией действия силы и точкой 0 называется силой плеча.

-прочность плеча от силы

Момент силы вокруг фиксированной оси Z называется скалярной величиной, равной проекции на ось крутящего момента, определенного относительно произвольной точки O оси Z . Если ось Z перпендикулярна плоскости, в которой векторы и, то есть совпадает с направлением, то момент силы, представленный как вектор, совпадает с осью

Ось, положение которой в пространстве остается неизменным при вращении вокруг тела при отсутствии внешних сил, называется свободной осью тела.
Для тела любой формы и с произвольным распределением массы через центр масс оси тела проходят 3 взаимно перпендикулярных оси, которые могут служить свободными осями: их называют главными осями инерции.

Находим выражение для вращательного движения тела. Пусть масса м твердая внешняя сила. Тогда работа этой силы за время d т составит

Возможна циклическая перестановка множителей в смешанном произведении векторов

Рабочее вращение тела — произведение момента силы на угол поворота тела d φ .Работа по увеличению его кинетической энергии:

Затем

или

  • основное уравнение динамики вращательного движения.

Если ось вращения совпадает с главной осью инерции через центр масс, то векторное равенство

I — главный момент инерции (момент инерции относительно главной оси)

§ 4 Угловой момент.Закон сохранения момента импульса

Угловой момент частицы A относительно фиксированной точки 0 — физическая величина, определяемая векторным произведением

— радиус-вектор из точки O в точку A

— импульс частицы.

— псевдовектор, его направление определяется правилом левой руки.

Угловой момент твердого тела вокруг фиксированной оси Z называется скалярной величиной, равной проекции на ось момента количества движения, определенной относительно произвольной точки O этой оси.Величина углового момента L z не зависит от точки O на оси Z .
Угловой момент твердого тела вокруг оси складывается из угловых моментов отдельных частиц:

Дифференциация по d т

— основное уравнение динамики вращательного движения.
Обычное векторное равенство

В замкнутой системе момент внешних сил равен нулю

Закон сохранения момента количества движения: момент количества движения замкнутой системы сохраняется, т.е. не изменяется во времени.


§ 5 Значения, характеризующие поступательное и вращательное движение, и взаимосвязь между ними:

Линейный

Угловой

Отношения

Временная зависимость

Путь (смещение)

вектор вращения

линейная скорость

угловая скорость

линейное ускорение

угловое ускорение

масса

м

момент инерции

I

импульс,

Импульс

п.

Угловой момент

L

усилие

ф

момент силы, крутящий момент

M

кинетическая энергия

Вт К,

кинетическая энергия вращения

Вт К,

начальная работа

дА,

элементарная работа вращательного движения

(PDF) Механика (на английском языке)

22

Разделение сил на «действующие» и «противодействующие» в третьем законе Ньютона

условно.Ошибочно искать в них причинную связь и считать, что «действующая» сила является причиной, а «противодействие

взаимодействует» — следствием. Обе силы возникают и исчезают в одно и то же время

.

Действующие и противодействующие силы имеют идентичную физическую форму.

. Если одно тело применяет силу на другом со стороны упругости или гравитации,

оно подвергается воздействию того же вида.

Тела могут взаимодействовать напрямую (контактируя друг с другом или примыкая)

или на расстоянии.Согласно современным представлениям, все взаимодействия

на расстоянии между телами осуществляются через материальные поля. Тело

создает в окружающем его пространстве определенное силовое поле, которое воздействует на другие тела

, находящиеся в этом пространстве. Поле — объективная реальность, с помощью

, взаимодействие которых передается через вакуум со скоростью света

с = 3.108м / с. Поле может существовать даже отдельно, независимо от поднявшего его тела

.Таково, например, электромагнитное поле.

После выключения передающей радиостанции поле не исчезает —

груша, а расширяясь, поднимает в приемных антеннах переменные токи.

Абсолютно пустого места нет. Пространство заполнено полями. Sub-

позиция и поле — два разных типа существования материи.

Силы, обусловленные контактным взаимодействием тел, такие как силы упругости

, давление, трение и другие, известны с древних времен

и, естественно, считались простыми и понятными.Гравитационные, электрические и магнитные силы, существование которых люди начали осознавать позже

, воспринимались как загадочные и непонятные. Это также причина того, что

физики XIX века пытались свести полевые взаимодействия

к контакту. На самом деле ситуация обратная. Причем основные полевые взаимодействия самые простые, а контактные — гораздо более сложные. Последние,

, а также химические, молекулярные, мышечные и другие силы, с которыми человек

в повседневной жизни сталкивается, даны следующим основным взаимодействиям: гравитационным, электро-недельным, сильным.

Первые два взаимодействия имеют теоретически неограниченный радиус ac-

. По этой причине все макроскопические явления вызваны этими взаимодействиями

.

Сильные взаимодействия несравнимо мощнее, но действуют с —

в микромире и характеризуются радиусом действия порядка ~

Почему механика является фундаментальной наукой: можно ли вывести динамику вращения из законов Ньютона?

Закон динамики вращения гласит, что момент инерции, умноженный на угловое ускорение, равен приложенному крутящему моменту.Это аналог 2-го закона Ньютона. Но можно ли логически вывести первое из второго?

Вращательный закон динамики иногда называют уравнениями Эйлера или вторым законом Эйлера , и он имеет форму d M / dt = T , где M — угловой момент, а T — крутящий момент / момент силы (как M , так и T — векторы). Первый закон Эйлера — это Второй закон Ньютона, d P / dt = F , где P — это импульс, а F — векторы силы.

Учебники по физике обычно предполагают, что уравнение Эйлера выводится из Второго закона Ньютона при сохранении углового момента. Стандартный вывод состоит в том, что парные внутренних сил в твердом теле, т. Е. Векторы сил между точками A и B твердого тела, а именно F ab и F ba , противоположны (по Ньютону). Третий закон) и центральный (что означает, что они действуют по прямой | AB |). Тогда их суммарный момент силы равен нулю: R A х F ab = — R B х F ba .Используя это вместе с сохранением углового момента и применяя некоторую векторную алгебру, учебники физики выводят уравнение Эйлера, d M / dt = T .

Использование сохранения углового момента здесь не представляет проблемы, потому что это фундаментальное следствие изотропии физического пространства на основе теоремы Нётер. Можно задать вопросы о ограниченном движении точек A и B в твердом теле. Определение твердого тела состоит в том, что расстояние между любыми двумя точками (например, точками A и B) остается постоянным.Это означает ограничение движения этих точек. Ограничения эквивалентны «виртуальным силам». Как эти виртуальные силы влияют на баланс углового момента, может быть предметом отдельного анализа.

Однако более важным является предположение, что силы действуют вдоль прямой линии. Физик Франк Вильчек отметил:

Когда большинство учебников обсуждают угловой момент, они вводят четвертый закон, согласно которому силы между телами направлены вдоль линии, которая их соединяет.Он вводится для того, чтобы «доказать» сохранение момента количества движения. Но этот четвертый закон совершенно неверен для молекулярных сил.

Вот что по этому поводу написал механик Клиффорд Трусделл:

«Достаточно ясно, что в статике равновесие моментов не обеспечивается равновесием сил, и наоборот. В динамике принцип момента количества движения развился поздно, и большая часть более ранних работ, касающихся его, создает впечатление, что эти два принципа каким-то образом надеялись быть эквивалентными, так что был бы только один закон движения.Эта иллюзия питается сегодня физиками, которые преподают механику.

… Эйлер и Даниэль Бернулли в своих ранних работах по жестким, связанным или гибким системам часто использовали принцип момента количества движения, часто замаскированный дополнительным квазистатическим предположением. Его важность стала осознаваться постепенно, так что только с трудом можно установить конкретную дату, когда она поднялась до уровня принципа механики. Я считаю, что правильной датой является 1751 год, год публикации статьи ЭЙЛЕРА 1744 года, которая была упомянута выше как первая, в которой «уравнения Ньютона» признаны достаточными, чтобы дать все механические принципы, определяющие движение сложной системы. Речь идет о натянутой нагруженной струне.В последней части этой же статьи получаются уравнения движения для системы из n жестких стержней, соединенных вместе и подверженных действию произвольных сил на стыках. Здесь недостаточно «ньютоновских уравнений». Кроме того, Эйлер устанавливает в качестве независимого принципа баланс момента количества движения относительно центра масс каждого стержня.

… Эти законы, которые можно назвать законами механики Эйлера, подразумевают не только «законы Ньютона» для материальных точек, но также и все другие принципы классической механики и столь же удобны для непрерывных тел, как и для дискретных систем.Первый закон эквивалентен «первым принципам» Эйлера 1750 года и дает общую теорему о центре масс. Второй закон в некоторых случаях следует из него; например, если все силы являются абсолютно непрерывными функциями массы и если все моменты являются моментами сил; но в более общих системах, таких как те, в которых присутствует напряжение сдвига, второй закон не зависит от первого.

Для твердого тела Эйлер смог напрямую и легко вывести свои прежние уравнения движения из второго закона.Закон момента импульса неуловим, его часто неправильно понимают даже сегодня.

В представлениях по механике для физиков он обычно выводится как следствие «законов Ньютона» для элементов массы, которые, как предполагается, притягиваются друг к другу с взаимными силами, которые являются центральными и попарно уравновешенными. Хотя формальные шаги этого вывода верны, результат слишком особенный для механики сплошных сред, а также методологически неверен для жесткой механики:

1.Никакие силы между частицами твердого тела по определению никогда не проявляются ни в каком движении. Для определения движения достаточно условия жесткости, примененного ко второму закону Эйлера. Предполагать взаимные силы — значит наслаждаться излишними причинами, которые должны быть устранены бритвой Оккама.
2. Введение взаимных сил в твердое тело ведет к злонамеренному действию на расстоянии в том случае, когда в этом нет необходимости.
3. В механике сплошной среды полная сила, действующая на конечное тело, возникает в основном из тензора напряжений, который представляет собой непрерывное действие материала на материал.Нет никакой физической причины предполагать, что силы возникают только в результате действия на расстоянии, и это не имеет никакой цели. Если механику твердого тела рассматривать как частный случай механики сплошной среды, напряжение внутри твердого тела неопределенно, и о нем не нужно упоминать, но единообразный процесс, основанный на втором законе Эйлера, остается возможным.

Ни одна работа Эйлера или любого другого ученого восемнадцатого века не приближается к твердым телам, предполагая силы, действующие на расстоянии, как в современных книгах.Метод открытия Эйлера молчаливо предполагает, что внутренних сил вообще нет; получается правильный ответ, но мы вправе сомневаться в том, что этот аргумент является достаточно общим. Его последнее обращение не делает никаких предположений или ограничений в отношении присутствия или характера внутренних сил ».

Другими словами, в механике есть три разных случая:

  1. Механика сплошной среды, где равновесие касательных напряжений не может быть выведено из равновесия нормальных напряжений.
  2. Статика твердых тел, где закон Архимеда рычага не может быть выведен из Третьего закона Ньютона, несмотря на попытки вывести его, используя соображения симметрии.
  3. Динамика твердого тела, где закон Эйлера может быть выведен из Второго закона Ньютона, предполагая, что в твердом теле есть внутренние силы и что они являются центральными.

Вопрос о том, рассматривать ли твердое тело или точечную массу как фундаментальный объект механики (это аргумент «бритвы Оккама» Трусделла выше) связан с проблемами, которые я обсуждал в одном из моих предыдущих записей в блоге.

Цель этой серии сообщений в блоге, которые я назвал «Почему механика — это фундаментальная наука», — показать, что в механике есть много фундаментальных логических проблем и, более того, что подход механики может отличаться от подхода физики. Ниже приводится определение одного из моих учителей, профессора Павла Жилина (мой перевод с русского):

« Механика — это не теория каких-либо природных явлений, а метод изучения природы. В основе механики нет законов, которые в принципе можно было бы опровергнуть экспериментально.Вместо этого существуют логические концепции, выражающие баланс определенных параметров, которые сами по себе недостаточны для построения автономных теорий и должны быть дополнены эмпирическими наблюдениями.

Другие записи в блоге этой серии:
1. Точечная масса и твердое тело
2. Являются ли законы Ньютона законами природы?
3. Логически ли статика предшествует динамике?
4. Можно ли вывести динамику вращения из законов Ньютона?

Темы редакции Gen Phys I

Предварительный текст

Темы редакции

(General Physics I)

  1. Вектор ускорения в движении по кривой: определение, разложение вектор ускорения на две составляющие (тангенциальную и нормальную), величина, направление и физический смысл каждого составная часть.Касательные и нормальные ускорения однородного круговое движение (UCM).
  2. Определение количества движения материальной точки и системы материальные точки. Вывод закона сохранения импульса для изолированная система. Пример применения закона. В Теорема об импульсе-импульсе. Значение понятия импульс.
  3. Определение импульса (для постоянной чистой силы и для общих случаи). Теорема об импульсе-импульсе. Смысл концепции импульса.
  4. Методика расчета работы, совершаемой постоянной силой и переменная сила. Теорема о работе кинетической энергии для случаев постоянные и переменные силы. Механическая энергия системы с / без рассеивания.
  5. Отношения между силой и потенциальной энергией. Уравнения, которые соотнести силу и энергию гравитационной силы (вблизи и вдали от поверхность Земли).
  6. Импульс материальной точки, системы материальных точек, сохранение импульса изолированной системы.Пример заявление.
  7. Центр масс. Движение системы частиц. Столкновение в система отсчета центра масс. Столкновения упругие и неупругие радиальные и нерадиальные. Радиальная абсолютно упругая и абсолютно неупругая столкновения.
  8. Момент инерции для систем дискретных точечных масс и для непрерывное распределение масс. Теорема о параллельной оси (включая его происхождение).
  9. Момент силы, действующей на вращающееся твердое тело (определение, компоненты вектора крутящего момента и их физический смысл).Вывод основного уравнения вращения твердого тела. вокруг фиксированной оси.
  10. Угловой момент материальной точки и твердого тела около фиксированная ось. Теорема углового момента и закон сохранение углового момента.

11. Определение простого гармонического движения (ПГД). Вывод дифференциальное уравнение для ШМ и его решение. Смысл параметры в решении. Вывод формул угловой частоты (периоды) простых и физических маятников.12. Энергия гармонического колебания. Затухающие колебания и энергия диссипация.

alexxlab

Добавить комментарий

Ваш адрес email не будет опубликован. Обязательные поля помечены *